Download as pdf or txt
Download as pdf or txt
You are on page 1of 45

P EVI S YEAR QUES I NS

(P ORKB KOF
INDIAN POLITY
AND
GOVERNANCE
For Civil. Services Rrelims Examination
'

(1100+ Topic Wise Solved Objective Questions from UPSC CSE Prelims (1991 to 2023),
CAPF & CDS EXAM (2014 to 2023} and STATE PCS Prelims Exams (1991 to 2023}}

PYQWORKBOOK
\ i\fi y I'
"c?o,,t..

NOTr~~
-
W..!

cs'

Forum Learning Centres


DELHI(ORN) MUKHERJEE PATNA HYDERABAD GURGAON
NAGAR
19, Pusa Road, 2nd 862,Banda 2nd floor, AG 1st Floor, SM Plaza, Property No. 894,
Floor, IAPL House, Bahadur Marg, Palace, E Boring RTC X Rd, Indira G.F., Saraswati
Opposite Metro Ist Floor, Dr. Canal Road, Patna, Park Road, Jawahar Vihar, Chakkarpur,
Pillar #95-96, Mukherji Nagar, Bihar 800001 Nagar, Hyderabad, Near MG Rd
Karol Bagh, New Near Batra Telangana- Metro Station,
Delhi-05 Cinema, Delhi - 500020 Sector-28,
110009 Gurgaon, Haryana
https:/ /blog.forumias.com Iwww.academy.forumias.com I 9311740400

KING R QUEEN P [ऋषभ राजपूत ]


PREVIOUS YEAR QUESTIONS (PYQ) WORKBOOK OF
INDIAN POLITY AND GOVERNANCE
For Civil Services Prelims Examination

First Edition (Print): 2024

No part of this publication may be reproduced or transmitted, in any form or by any means,
electronic, mechanical, photocopying, recording or otherwise, or stored in any retrieval system of
any nature without the written permission of the copyright holder and the publisher, application
for which shall be made to the publisher.
©Forum (Flaviant Network Pvt. Ltd.)

This book/ material is sold subject to the conditions that it shall not, by way of trade or otherwise,
be lent, re-sold, hired out or otherwise circulated without the publisher's prior consent in any
form of binding or cover other than that in which it is published and without a similar condition
including this condition being imposed on the subsequent purchaser.

We are continuously working on improving our quality. If you are unhappy or happy with the
quality of this study material, want to report an error, or want to leave us feedback, you can email
us at ravi@forumias.com

© 2024 1st Print Edition I For feedback email us : :ravi@forumias.com

KING R QUEEN P [ऋषभ राजपूत ]


The PYQWORKBOOK
Dear Aspirants,

We at ForumIAS have come up with a PYQWorkbook series to provide the aspirants with a
holistic idea regarding challenges associated with the changing pattern and difficulty level
of the UPSC Prelims Exam. The PYQWorkbook includes the questions asked indifferent
objective-based exams conducted by Union Public Service Commission like UPSC CSE
Prelims Exam, CAPF Exam, and CDS Exam. It also includes the solved previous year's
questions from different State PCS Prelims Examinations to equip the aspirants with every
relevant fact and concept needed for their UPSC CSE Prelims exam preparation. These
workbooks are divided subject wise. The PYQ Workbook Series, along with ForumIAS's
Practicebooks and Full-Length Testbooks, will help in building conceptual clarity with
respect to different subjects as well as understanding the importance/relevance of each
topic.

Indian Polity and Governance is one of the pillars of UPSC CSE Prelims exam from
which there has been a significant number of questions each year which is evident from
the graph provided below:

POLITY

"S I I 14 I I
0 10- - - - _I - - - - - - - - - - - - - - - I_I - - - - - - _I_ - - - - - - - JI - - - - - - - ,12 - - - - _I - -
Z 10 I 1·0 I , I
I . 6 I I I I
5 - - - - - _I - - - - - - - - .1 - - - - - - - - - - - - - - - _I - - - - - - - - .l - - - - - - - - L - - - - .J - -
2012 2014 2016 2018 2020 2022 2023
Year

Number of Indian Polity and Governance questions asked in UPSC CSE GS -1 Prelims
from 2011 till 2023

Regarding the Indian Polity and Governance part, the focus has remained on the core topics
like Parliament, Executive, Judiciary, Fundamental Rights, DPSPs and Constitutional
Amendments etc. Though in last few years some shift has been seen:

KING R QUEEN P [ऋषभ राजपूत ]


* The Governance aspect being given more weightage like Questions on Tea board etc.
* Not just knowing the concept but a deeper understanding of the Constitution and its
provisions.
* A few laws related questions like judicial custody, parole, preventive dentation, Prevention
of Terrorism Act, 2002 and The Arms (amendment) Act, 2019 etc.
* Important judgements of Supreme Court (both in Mains as well as Prelims).
* 1-2 Political Science type questions like due process oflaw and Constitutionalism etc.

But overall, the serious aspirants have been able to score more than 70% from this section
and have been able to dear UPSC CSE Prelims based upon that. Overall conceptual clarity as
well as basic reading of newspaper articles related to this section is simply needed.

The compiled previous year UPSC Questions matched with the official answer key would
help the aspirants in aligning their preparation as per the demand of the exam.

Happy Learning and Happy Testing!!!


Yourteam@ForumIAS

KING R QUEEN P [ऋषभ राजपूत ]


,, !I
•,'

,, !/
II II
administration of Scheduled Areas and Scheduled Tribes. It Important Tip
outlines the powers and functions of the tribal advisory councils,
The Citizenship Act, 1955 was amended 6 times in
safeguards for the welfare and advancement of Scheduled Tribes,
1986, 1992, 2003, 2005, 2015 and 2019.
and other related provisions.
The CAA 2019 provides citizenship on the basis
4 th pair is incorrectly matched: The Ninth Schedule of the of religion to six undocumented non-Muslim
Constitution of India a special provision that was added to communities (Hindus, Sikhs, Buddhists, Jains,
protect certain laws from judicial scrutiny on the grounds of Parsis and Christians) from Pakistan, Afghanistan
violating fundamental rights. It includes laws related to land and Bangladesh who entered India on or before 31st
reforms, the abolition of the zamindari system, and other December, 2014.
social and economic reforms. It exempts the members of the six communities from
any criminal case under the Foreigners Act, 1946 and
109. Solution: (c)
the Passport Act, 1920.
Exp) Option c is the correct answer.
Statement 1 is correct- Equality entails the absence of 112. Solution: (a)
privileges or discrimination within society, and the Exp) Option a is the correct answer.
Preamble emphasizes equality of status and opportunity for The doctrine of independence is not to be raised to the
all citizens. level of a dogma so as to enable the judiciary to function
Statement 2 is correct-Through the 42nd Amendment Act as a kind of super-legislature or super-executive.
of 1976, the phrase was added to the Preamble to emphasize The judiciary is there to interpret the Constitution or
the importance of maintaining the unity and integrity of adjudicate upon the rights. Option 1 is correct.
India as a sovereign, socialist, secular, and democratic It is the great tribunal which has to draw the line between
republic. liberty and social control. Option 2 is correct

Important Tips 113. Solution: (d)


The Indian Constitution's Preamble is derived from the Exp) Option d is the correct answer.
US Constitution and is often referred to as the "identity 1. Sixth schedule to the constitution of India provides for
card" of the Constitution. setting up of autonomous district council (option d is
It is based on the Objective Resolution proposed by correct)
Pandit Nehru and was amended once through the 42nd 2. The Constitution of India makes special provisions for the·
Constitutional Amendment Act, 1972, to include the
administration of the tribal dominated areas in four states
words "Socialist, Secular and Integrity."
viz Assam, Meghalaya Tripura and Mizoram.
The Preamble is non-justiciable and does not grant
3. As per Article 244 and sixth Schedule these areas are called
legislative power or restrict it. It is an integral part of the
"Tribal Areas
Constitution and can be amended by a simple majority
under Article 368. It was added by the Constituent 4. The Sixth Schedule envisages establishment of Autonomous
Assembly after the enactment of the rest of the District Councils (ADC)
Constitution. :
Tribal areas in the four states
110. Solution: (b) Assam North Cachar Hills District
Exp) Option b is the correct answer The Karbi Anglong District
The Preamble of the Constitution oflndia was amended in 1976 The Bodoland Territorial Areas District
by the 42nd Constitutional Amendment Act. This amendment Meghalaya Khasi Hills District
introduced three terms to the Preamble: Socialist, Secular, Jaintia Hills District
and Integrity. The term "Fraternity" was already a part of the
The Garo Hills District
original Preamble and was not introduced or amended in
Tripura Tripura Tribal Areas District
1976.
Mizoram The Chakma District
111. Solution: (b) The Mara District
Exp) Option b is the correct answer
114. Solution: (d)
Article 11 of the Indian Constitution grants the Parliament
the authority to regulate citizenship rights, and it is under this Exp) Option d is the correct answer.
provision that the Citizenship (Amendment) Act (CAA) was The government of India act 1858 is also known as the
enacted. As Article 11 is part of Part II of the Constitution of "act for better government of India".
India, the CAA can be classified within the framework of Part II It changed the designation of governor general of India
of the constitution. to that of viceroy of India.

PYQ W o r k b o o k - - - - - - - - - - - - - - - - - - - - - - - - - - - - - - - - - - - - - - 1 -

KING R QUEEN P [ऋषभ राजपूत ]


II II
The administration of India was taken from the East 116. Solution: (d)
India Company and placed directly under the British Exp) Option dis the correct answer
Crown. (option l is correct) The Indian Constitution is unique in its contents and
Thus the country was kept under the direct rule of the spirit.
British and the Cabinet. Though borrowed from almost every Constitution of the
A new post called Secretary of State was added who world, the Constitution of India has several features that
would be directly responsible to British cabinet and also distinguish it from the Constitutions of other countries.
a part of it. Independence of Judiciary has been taken from the
The act also ended the double government which was Constitution of USA.
started by the Pitts India act 1784. Following features have been borrowed by the British
Constitution:
A 15 member advisory council to the secretary of state
was formed. Britain 1. Parliamentary System of government
This act was dominated by the principle of absolute 2. Rule of Law
imperial control without any popular participation in . 3. Legislative procedure
administration of the country. ( Option 3 is correct) • 4. Single citizenship
Under this act, all the powers of Crown were exercised by 5. Cabinet system
Secretary of State of India and was assisted by Council of
6. Prerogative writs
India (Council of fifteen members).
7. Parliamentary privileges
TJnder the Pitts India .i\.ct of 1784 for the first tiiT1e,
the British Government was given supreme control 8. Bicameralism
over Company's affairs and its administration in India. United 1. Impeachment of the president
States of 2. Functions of president and vice-president
(Option 2 is incorrect)
America
3. Removal of Supreme Court and High court
115. Solution: (d)
judges
Exp) Option d is the correct answer.
4. Fundamental Rights
Indian Councils Act of 1861 made a beginning of
5. Judicial review
representative institutions by associating Indians with
the law-making process. • 6. Independence of judiciary
7. The preamble of the constitution
It thus provided that the viceroy should nominate some
Indians as non-official members of his expanded council. 117. Solution: (b)
(Statement l is correct) Exp) Option bis the correct answer.
In 1862, Lord canning the then viceroy nominated three Ninth Schedule says that the article mentioned in this are
Indians to his Legislative council immune from judicial review it is for the larger good of
It empowered the viceroy to make rule and orders for the the society, the government cannot be challenged in the
more convenient transaction of business in the council. court for his actions.

It also gave recognition to the Portfolio system introduced Ninth Schedule was added to the Constitution by First
Constitutional Amendment, 1951.
by Lord canning in 1859
In a landmark ruling on 11th January 2007, the
Under this a member of the viceroy's council was made
Supreme Court ruled that all laws (including those in
in charge of one or more departments of the government the Ninth Schedule) would be open to judicial review,
and was authorized to issue final orders on behalf of the if they violated the basic structure of the Constitution.
council on matters of his department(s). (Statement b is incorrect)
The Governor-General also had the power to promulgate
118. Solution: (b)
ordinances without the council's concurrence during
emergencies (Statement 3 is incorrect). Exp) Option bis the correct answer.
On 26th November 1949, 3 provisions i.e .. , Elections,
The non official members of the Viceroys Executive
citizenship and provisional Parliament came into force
council did not take part in the discussions and their
and rest of the provisions only on 26th January 1950.
role was only advisory.
Fundamental Rights of the citizen came into force on
The non-official members of the Executive Council were 26th january 1950.
not interested in attending the meetings of the Council,
moreover, under this Act they were not bound to attend 119. Solution: (a)
them either.The Indian members were not eligible to Exp) Option a is the correct answer.
oppose any bill and most often the bills were passed in one As per Artide-344(1) and 351 of Indian Constitution,
sitting without discussion. (Statement 2 is incorrect) the 8th Schedule includes 22 language.

1 1 1 > - - - - - - - - - - - - - - - - - - - - - - - - - - - - - - - - - - - - P Y Q Workbook

KING R QUEEN P [ऋषभ राजपूत ]


II INDIANJ!OLITY ANO GOVERNANCE

The Eighth Schedule to the Constitution consists of the by virtue of article 6 or article 8, if he has voluntarily
following 22 languages: acquired the citizenship of any foreign State.
Of these languages, 14 were initially included in the As per article 11 The Parliament has the right to
Constitution. make any provision concerning the acquisition and
Sindhi language was added by the 21 st Amendment Act termination of citizenship and any other matter relating
of 1967. to citizenship.
Konkani, Manipuri, and Nepali were included by the 123. Solution: (c)
71 st Amendment Act of 1992.
Exp) Option c is the correct answer.
Bodo, Dogri, Maithili, and Santhali were added by the
The 9th schedule was added to the constitution by the
92 nd Amendment Act of 2003 which came into force in
1st amendt. to the constitution in 1951 (Statement 3 is
2004. incorrect)
120. Solution: (d) The Ninth Schedule of the Constitution contains a list of
Exp) Option d is the correct answer. central and state laws which cannot be challenged in
courts. (Statement 4 is incorrect)
Article 25 says that all persons are equally entitled to
freedom of Conscience and the right to freely profess, 124. Solution: (b)
practice and propagate religion.
Exp) Option bis the correct answer.
The Indian Constitution embodies the positive concept
Democracy is a form of government in which the rulers
of secularism ie, all religions in our country (irrespective
are elected by the people.
of their strength) and no official status was given to
certain religions. It is based on a free and fair election where these
currently in a power having a free chance of loosing.
42nd amendment act of 1976, the Preamble to the
constitution of India asserted that India is a secular In a democracy, each adult citizen have one vote and
nation. each vote have one value and the decision making
power rests with those elected by the people.
Secularism is a belief system which rejects religion, or
the belief that religion should not be part of the affairs of Democracy also enhances the dignity of citizens.
the state. (Statement 2 is incorrect) Abraham Lincoln had said "Democracy is the
government of the people, by the people and for the
121. Solution: (d) people".
Exp) Option d is the correct answer.
125. Solution: (b)
The members of constituent assembly were chosen indirectly
by members of provincial assemblies, who in turn were Exp) Option b is the correct answer.
elected through Provincial Election of 1946. (Statement 1 BR Ambedkar has emerged as the epitome personality of
is correct) modern India.
The constituent assembly had representative of princely He was appointed as the Chairman of the constitution
states , who were nominated by the princes. (Statement 2 is drafting committee in 1947.
incorrect) He is recognised as the "Father of the Constitution of
The constituent assembly had representation from all parts India".
of India , every community, every section of the society was Major contributions of Ambedkar to modern India :
represented in the assembly and hence the discussions in
a) Education
the constituent assembly was influenced by public opinion
over an issue, although rational decision was taken after fair b) Changing hierarchical Structures of Indian society
discussions. (Statement 3 is incorrect) c) Bringing untouchables to the mainstream of Indian
To create a sense of collective participation the public was society.
asked to send in their views . Many of the linguistic minorities d) Framing an inclusive and secular Constitution.
wanted the protection of their mother tongue, religious e) Upliftment of marginalised
minorities asked for special safeguards, while dalits demanded f) Fundamental Rights
an end to all caste oppression and reservation of seats in
government bodies. Important issues of cultural rights and g) Economic Contributions
social justice raised in these public discussions were debated 126. Solution: (d)
on the floor of the Assembly. (Statement 4 is correct)
Exp) Option d is the correct answer.
122. Solution: (c) In Part II of Constitution under Article 5-11, citizenship
Exp: Option c is the correct answer. clause has been explained.
As per article 9 No person shall be a citizen of India by Full civil and political rights are entitled to the citizen
virtue of article 5, or be deemed to be a citizen of India of India.

PYQ W o r k b o o k - - - - - - - - - - - - - - - - - - - - - - - - - - - - - - - - - - - - - 1 1 1
l]F'orurn~

KING R QUEEN P [ऋषभ राजपूत ]


II INDIAN PQUTY i\ND GOVERNANCE II
Also, they are entitled to be a Member of Parliament or
1. The Preamble is based on 'Objective Resolution' by
State Legislature but one has to also satisfy the age limit
Nehru and was adopted and enacted by the Constituent
criterion in order to be an MP or MLA.
Assembly of India on 26th November 1949.
Citizens of India has right of suffrage for election to the
2. It has been amended only once through the 42nd
House of People (of the Union) and legislative assembly Amendment Act of 1976, which added the words
of every state. "Secular" and "Socialist" to the Preamble.
Important Tips 3. The Preamble serves as a guiding light for the
1. Citizenship is listed in the Union List. government, lawmakers, and citizens in shaping the
nation's policies and actions.
2. The term 'Citizen' is not defined in the constitution.
3. Citizenship of India can be acquired in the following 129. Solution: (b)
ways: Exp) Option bis the correct answer.
Citizenship at the commencement of the Constitution The Constitution of India provides for the division of
Citizenship by birth powers between central and state government. The seventh
Citizenship by descent schedule provides for three list- State List, Central List, and
Concurrent List. Out of the three list, the Concurrent list
Citizenship by registration
includes subjects on which both the Parliament (Central
Citizenship of naturalization government) and State Legislature (state governments)
By incorporation of territory (by the Government of can legislate independently and concurrently.
Tndia)
Important Tips
4. Termination of the Citizenship is possible in three
Few of the subjects under the Central List:
ways:
1. Defense and armed forces
Renunciation, Termination, Deprivation
2. Foreign affairs and international relations
127. Solution: (a) 3. Atomic energy
Exp) Option a is the correct answer. 4. Currency, coinage, and legal tender
Somnath Lahiri was a member of the Constituent Assembly 5. Railways
of India and a prominent leader of the Communist Party 6. Airports and airways
of India. He was a vocal critic of the British rule in India 7. Interstate trade and commerce
and he believed that the Constituent Assembly was not
8. Banking and financial institutions
truly representative of the Indian people. In a speech in
the Constituent Assembly on 19 December 1946, Lahiri said 9. Taxes on income other than agricultural income
that the Constituent Assembly was "British made" and that it Few of the subjects under the State List:
was "working the British plan as the British should like it to 1. Police and public order
be worked out". He argued that the Assembly was dominated 2. Public health and sanitation
by the Indian National Congress, which was a party that had 3. Agriculture and irrigation
close ties to the British government
4. Fisheries
128. Solution: (d) 5. Local government
Exp) Option d is the correct answer. 6. Public works
The preamble to the Constitution of India is a short 7. Public entertainment
introductory statement that sets out the basic principles 8. Land and land revenue
and objectives of the Constitution. It is not enforceable in 9. Education at the state level
a court of law, but it is an important source of guidance for
Few of the subjects under the Concurrent List:
interpreting the Constitution. The Supreme Court has held
that the preamble can be used to interpret the Constitution I. Criminal law and procedure
in cases where the language is ambiguous. 2. Marriage and divorce
3. Adoption
Important Tip
4. Bankruptcy and insolvency
Important points about Preamble:
5. Education, including technical education
Key elements of Preamble:
Sovereign, socialist, secular, and democratic and 6. Population control and family planning
republic. 7. Protection of wildlife and animals
Justice, liberty, equality, and fraternity. 8. Trusts and trusteeship
Unity and integrity of the nation. 9. Price control and essential commodities

llll,--------------------------------------PYQWorkbook

KING R QUEEN P [ऋषभ राजपूत ]


II. INDIAN POLITY AND GOVERNANCE II
130. Solution: (c) 134. Solution: (b)
Exp) Option c is the correct answer. Exp) Option b is the correct answer
Statement l is correct- Parliament can legislate a subject The Independence Act, 1947, was a British Act of Parliament
under the State list in certain conditions. These conditions that partitioned British India into two independent
include during a national emergency under Article dominions, India and Pakistan. The Act did not lay down
250, when a state makes a request under Article 252, to a constitution for either dominion. Instead, it provided
implement international agreements under Article 253, and for the establishment of constituent assemblies in each
dominion, which would be responsible for framing their
during President's Rule under Article 356.
own constitutions.
Statement 2 is incorrect- Parliament can legislate a subject
under the State list under certain circumstances during 135. Solution: (b)
National Emergency and President's rule. So, the statement Exp) Option b is the correct answer
that Parliament cannot legislate a subject under the State list The Dhar Commission was appointed by the Government
under any circumstances is not correct. of India in 1948 to study the feasibility of reorganizing the
Statement 3 is correct- As per Article 249 of the Constitution, states of India on a linguistic basis.
if the Rajya Sabha passes a resolution supported by not less Important Tips
than two-thirds of the members present and voting, stating
Dhar Commission-Recommendations
that it is necessary or expedient in the national interest,
1. It advocated for state reorganization based on
Parliament can make laws on any matter enumerated in the
administrative convenience rather than linguistic
State list.
considerations.
131. Solution: (b) 2. The Commission recommended the reorganization of
Exp) Option b is the correct answer States on the basis of the following criteria:

Pair 1 matched correctly: Public health and sanitation are Geographical contiguity
among the subjects listed in the State List. Financial self-reliance
Pair 2 matched correctly: Citizenship, naturalization, and Administrative viability
aliens are among the subjects listed in the Union List. Potential for development
Pair 3 matched correctly: Legal, medical, and other 136. Solution: (d)
professions are among the subjects listed in the Concurrent
Exp) Option d is the correct answer.
List.
The Constituent Assembly of India was the body that framed
132. Solution: (b) the Constitution of India. It was established under the
Exp) Option b is the correct answer Cabinet Mission Plan of 1946 and consisted of 389 members,
of which 296 were elected by the provincial assemblies and
Statement 1 is incorrect: The 'Objectives Resolution' was
93 were nominated by the princely states
actually moved by Jawaharlal Nehru on December 13, 1946,
Statement 1 is correct: The Constituent Assembly was not
in the Constituent Assembly of India. This resolution laid
based on an adult franchise, as the provincial assemblies
down the fundamental principles and objectives that would
themselves were elected on a limited franchise
guide the framing of the Constitution.
Statement 2 is incorrect: It was also not a result of direct
Statement 2 is correct: Upon India's independence on 15 election, as the members were elected indirectly by the
August 1947, the new prime minister Jawaharlal Nehru provincial assemblies or nominated by the princely states.
invited Ambedkar to serve as the Dominion of India's Law
Statement 3 is correct: It was a multi-party body, as it
Minister; two weeks later, he was appointed Chairman of
included representatives from various political parties
the Drafting Committee of the Constitution for the future such as the Indian National Congress, the Muslim League,
Republic of India. the Communist Party of India, etc.
133. Solution: (a) Statement 4is correct: It worked through severalcommittees,
such as the Drafting Committee, the Fundamental Rights
Exp) Option a is the correct answer
Committee, the Union Powers Committee, etc., to deal
The Constitution of India was framed between December with different aspects of the constitution-making process .
1946 and December 1949. The Constituent Assembly, which
was elected in 1946, met for the first time on December 137. Solution: (b)
9, 1946. The Assembly appointed a Drafting Committee in Exp) Option b is the correct answer.
1947, and the final draft of the Constitution was adopted According to Granville Austin, one of the most significant
on November 26, 1949. The Constitution came into force on contributions of India to constitution-making was
January 26, 1950. Consensus and Accommodation. This is because he argued

PYQ W o r k b o o k - - - - - - - - - - - - - - - - - - - - - - - - - - - - - - - - - - - - - - - - 1 l a J I
mFon,n11Ji\i

KING R QUEEN P [ऋषभ राजपूत ]


II INDIAN POU'J;'Y A1'fD GOVE~NANGE II
that the Indian Constitution was a product of a remarkable diminish the area of any State;
process of consensus-building and accommodation among
alter the boundaries of any State;
various groups and interests in the Constituent Assembly and
outside. He highlighted how the framers of the Constitution alter the name of any State.
sought to reconcile the conflicting demands of democracy, 139. Solution: (a)
nationalism, social justice, federalism, secularism, minority
Exp) Option a is the correct answer.
rights, and individual liberties in a diverse and complex
society. Statement l is correct: The Preamble of the Constitution
of India is a brief introductory statement that sets out the
Important Tips guiding principles and purpose of the Constitution. It is
Granville Austin (1927-2014) was an American based on the Objective Resolution, which was moved by
historian and a leading authority on the Indian Jawaharlal Nehru in the Constituent Assembly on 13
Constitution. December 1946 and adopted on 22 January 1947.
He wrote two influential books on the making and Statement 2 is correct: The idea of the preamble was
working of the Indian Constitution, namely The borrowed from the Constitution of the USA.
Indian Constitution: Cornerstone of a Nation (1966) Statement 3 is correct: The Preamble declares India to be a
and Working a Democratic Constitution: The Indian sovereign, socialist, secular, democratic republic and secures
Experience (1999). to all its citizens justice, liberty, equality and fraternity.
He was awarded the Padma Shri by the Government The words socialist and secular were added by the 42nd
of India in 2011 for his contribution to constit11tional Amendment in 1976.
studies. Statement 4 is incorrect: While the Preamble of the
Constitution of India sets out the ideals and objectives
138. Solution: (c)
of the Constitution, it does not explicitly mention the
Exp) Option c is the correct answer. Fundamental Rights. The Fundamental Rights are enshrined
According to Article 3 of the Constitution of India, the in Part III of the Constitution.
Parliament may by law:
140. Solution: (a)
Alter the boundaries of any State: The Parliament has
Exp) Option a is the correct answer.
the power to alter the boundaries of any State in India.
This can be done through the process of reorganization Statement l is correct: Constitutional government is a form
of states, which req11ires the introd11ction and passing of of limited government. The constitution sets the boundaries
a law by Parliament. and limits of the powers of the government and prevents it
from becoming arbitrary or tyrannical. The constitution also
Increase the area of any State: The Parliament also has
ens11res that the government is accountable to the people and
the authority to increase the area of any existing state by
respects their rights and freedoms.
annexing or merging territories with it. This can be done
through the enactment of a law by Parliament. Statement 2 is correct: A constitutional government is one
where the constitution is the basis of public authority.
Statement I is not related to Article 3, b11t to Article 253, The constitution establishes the structure, functions, and
which empowers Parliament to make laws for implementing procedures of the government and defines its relationship
treaties, agreements or conventions with foreign countries or with the people. The constitution also lays down the
international organizations. principles and values that guide the actions and decisions of
Statement 4 is not related to Article 3, but to Article 244A, the government and its officials.
which provides for the creation of an autonomous state Statement 3 is incorrect: A constitutional government does
comprising certain tribal areas in Assam and for the creation of not necessarily have a monarch as the formal head of the
local legislature or council of ministers or both for such state. state. A constitutional government can have different forms of
Important Tips executive leadership, such as a president, a prime minister, or
a chancellor.
Article 3 of the Constitution of India deals with the
formation of new States and alteration of areas, boundaries Statement 4 is incorrect: A constitutional government does
or names of existing States. In this regard, the Parliament not always have a universal adult franchise. A constitutional
may by law: government can have different forms of electoral systems,
such as proportional representation, first-past-the-post, or
form a new State by separation of territory from any
mixed-member proportional. A universal adult franchise is
State or by uniting two or more States or parts of States,
one of the possible ways of ensuring political participation
or by uniting any territory to a part of any State;
and representation in a constitutional democracy, but it is
increase the area of any State; not a necessary condition for a constitutional government.

■ IHI

1 1 1 - - - - - - - - - - - - - - - - - - - - - - - - - - - - - - - - - - - - - P Y Q Workbook

KING R QUEEN P [ऋषभ राजपूत ]


Article 19 - Protection of six rights related i:o freedom - 168. Solution: (a)
(a) of speech and expression; (b) to assemble peaceably Exp) Option a is the correct answer.
and without arms; (c) to form associations or unions; The Constitution of India was amended by the Eighty-
(d) to move freely throughout the territory of India; (e) sixth Amendment Act of 2002 (and not the Eighty-fifth
to reside and settle in any part of the territory of India;
Amendment Act) to include Article 21A, which mandates
and (f) to practice any profession, or to carry on any
that every state must ensure free and compulsory education
occupation, trade or business.
for children aged six to fourteen. This amendment recognizes
Article 29 - Protection of language, script and culture education as a Fundamental Right protected under Part III
of minorities. of the Constitution.
Article 30 - Right of minorities to establish and
administer educational institutions. 169. Solution: (b)
Exp) Option b is the correct answer.
166. Solution: (a)
Statement 1 is correct- The procedure for amending the
Exp) Option a is the correct answer.
Constitution is provided in Article 368 of the Constitution of
The Constitution of India includes provisions in Part IV India. According to article 368, constitution can be amended
(Article 36-51) known as the Directive Principles of State
in three ways- Simple Majority, Special Majority, Special
Policy (DPSP).
Majority of the Parliament and the ratification of half of the
One such principle is highlighted in Article 50, which state legislatures.
emphasizes the importance of separating the judiciary
Statement 2 is correct- A bill to amend the Constitution
from the executive in the public services of the State. This
can be introduced in either House of Parliament. It can be
provision reflects the commitment to maintaining the
introduced as a Constitutional Amendment Bill and must go
independence and impartiality of the judiciary, ensuring a
through the prescribed legislative process before it can be
fair and just system of governance.
enacted into law.
Important Tipss
Statement 3 is correct- The special procedure outlined in
Classification of Directive Principles of State Policy:
Article 368 of the Constitution vests constituent powers upon
Socialistic Principles - Article 38, 39, 39A, 41, 42, 43,
43A, 47 the ordinary legislation. This means that the Parliament,
Gandhian Principles- Article 40, 43, 43B, 46, 47, 48 while acting in its legislative capacity, has the power to amend
Liberal-Intellectual Principles- Article 44, 45, 48, or modify the provisions of the Constitution provided that
48A, 49, 50, and 51. Basic Structure of the Indian Constitution remains intact.
The Constitution does not contain any classification of
Directive Principles. It is classified on the basis of their 170. Solution: (c)
content and direction Exp) Option c is the correct answer.
167. Solution: (d) A national emergency in India can be declared based on the
Exp) Option d is the correct answer. grounds of war, external aggression, or armed rebellion. The
term "armed rebellion" was introduced through the 44th
Article 12 of the Constitution of India defines the term
amendment to the Constitution. Prior to this amendment, it
'State' for the purpose of interpreting fundamental rights.
was referred to as "internal disturbance."
According to Article 12, the term 'State' includes not only
the Government oflndia and the Parliament oflndia but also Important Tips:
the Governments and Legislatures of each State. A national emergency in India can be proclaimed
Important Tips: under Article 352 of the Constitution during situations
of war, external aggression, or armed rebellion that
According to Article 12, 'State' includes:
poses a threat to the country's security and integrity.
Government and Parliament of India- executive and
The President of India has the authority to declare a
legislative organs of the Union government.
national emergency based on the advice of the Union
Government and legislature of states- executive and Cabinet.
legislative organs of state government.
Once a national emergency is proclaimed, the
All Local Authorities- municipalities, panchayats, central government assumes additional powers, and
district boards, improvement trusts, etc. the federal structure of the country undergoes a
• All Other Authorities- statutory or non-statutory temporary change.
authorities like LIC, ONGC, SAIL, etc. Fundamental rights guaranteed by the Constitution can
A private body or an agency working as an instrument be suspended or curtailed during a national emergency,
of the State falls within the meaning of the 'State' under except for certain rights mentioned in Article 20 and
Article 12. Article 21.

PYQWorkbook---------------------------------
&IForumla

KING R QUEEN P [ऋषभ राजपूत ]


II INDIAN POLITY AND GOVERNANCE II
The proclamation of a national emergency must be and ensure that every child in India has access to quality
approved by both houses of Parliament within one education during the crucial formative years of their life.
month and is valid for six months. It can be extended Important Tips:
with the approval of Parliament in subsequent intervals
Article 21 A (Fundamental Right)- The State shall
of six months.
provide free and compulsory education to all children
171. Solution: (d) of the age of six to fourteen years in such manner as
the State may, by law, determine.
Exp) Option d is the correct answer.
Article 45 (Directive Principle of State Policy)- The
Freedom of speech and expression in India is not absolute and
State shall endeavor to provide early childhood care
has reasonable restrictions. Restrictions include defamation,
and education for all children until they complete the
indecency, morality, and incitement to offense. These
age of six years.
limitations aim to protect reputation, maintain decency, and
Article 51 A (Fundamental Duties) - Parent or guardian to
prevent illegal activities. The Constitution guarantees this
provide opportunities for education to his child or, as the case
fundamental right while allowing necessary limitations for
may be, ward between the age of six and fourteen years.
public safety.
174. Solution: (a)
172. Solution: (a)
Exp) Option a is the correct answer.
Exp) Option a is the correct answer
Article-20 of the Constitution of India protects a person
Articles 31 A and 3 IC are exceptions to the Fundamental
against- double jeopardy. Double jeopardy refers to the
Rights enumerated in Article 14 and Article 19, as they ailow
principle that an individual cannot be prosecuted or
for certain restrictions on equality and property rights in
punished more than once for the same offense. The article
specific contexts for the purpose of agrarian reforms and
states three specific protections:
socioeconomic welfare.
No Double Jeopardy.
Important Tips: No ex-post facto law.
Article 31A: It saves five categories of laws from being No self-incrimination.
challenged and invalidated on the ground of contravention
of the fundamental rights conferred by Article 14 and Important Tips:
Article 19. It includes: Double Jeopardy- Double jeopardy protection is
Acquisiti.J;1 ,>f estates and related rights by the State; applicable only in legal proceedings before a court or
judicial tribunal. It does not apply to proceedings before
Taking over the management of properties by the State;
departmental or administrative authorities since they are
Amalgamation vf corporations;
not of a judicial nature.
Extingu;shr.·1e11t or modification of rights of directors
No ex-post facto law- An ex-post-facto law refers to a
or shareholders of corporations
law that applies punishments or increases penalties for
Extinguishmcnt or moo11ication of mining leases. actions that have already been committed in the past. In
It also provides the guaranteed right to compensation in other words, it is a law that is enforced retroactively. It is
case of acquisition or requisition of the private property applicable only to criminal laws.
by the state. No Self-Incrimination- It means no person accused of any
Article 31 C: It contained two provisions: offence shall be compelled to be a witness against himself.
It says that no law that seeks to implement socialistic The protection against self-incrimination extends to both
directive principles specified in Articles 39 (b) and (c), oral evidence and documentary evidence. It extends only
shall be declared void on the grounds of contravention to criminal proceedings. However, it does not extend to
of the fundamental rights conferred by Article 14 or (i) compulsory production of material objects,
Article 19. (ii) compulsion to give thumb impression, specimen
Moreover, no law containing a declaration that it is for signature, blood specimens, and
giving effect to such policy shall be questioned in any (iii) compulsory exhibition of the body.
court on the ground that it does not give effect to such
a policy. 175. Solution: (c)
Exp) Option c is the correct answer.
173. Solution: (b)
During the debates of the Constituent Assembly, Dr. B.R.
Exp) Option b is the correct answer. Ambedkar, the principal architect of the Indian Constitution,
The Constitution (86th Amendment) Act, 2002 added the defended Article-356 on the plea that it would be used as "a
fundamental duty of providing opportunities for education matter of last resort." He argued that this provision would
to one's child between the age of 6 and 14 years. This only be invoked in exceptional circumstances when all
amendment aimed to emphasize the importance of education other remedies and alternatives have been exhausted. The

B I - - - - - - - - - - - - - - - - - - - - - - - - - - - - - - - - - - - - - P Y Q Workbook

KING R QUEEN P [ऋषभ राजपूत ]


II INDIANPOLITYAND GOVERNANCE II
intention behind Article-356 was to ensure the maintenance Defended and guaranteed by the Supreme Court.
of constitutional order and stability in the country by
There are six fundamental rights recognised by the
providing a mechanism to address critical situations in the
Indian constitution:
states.
Right to Equality (Article 14-18)
Important Tips: Right to Freedom (Article 19-22)
President's Rule, also known as State Emergency, is the Right against Exploitation (Article 23-24)
suspension of the state government and the imposition
Right to Freedom of Religion (Article 25-28)
of direct central government rule in a state of India.
Cultural and Educational Rights (Article 29-30)
It is implemented under Article 356 of the Indian
Constitution when the President deems that the Right to Constitutional Remedies (Article 32)
state's governance cannot be carried out according to NOTE: Originally, there were 7 Fundamental Rights
constitutional provisions. enshrined in the constitution. However, Right to
The decision to impose President's Rule is taken by Property was deleted as a fundamental right through the
the President on the advice of the Governor of the 44th Amendment Act, 1978. It was made a legal right
concerned state or the Union Cabinet. (Article 300A).
The state legislative assembly is either dissolved or 178. Solution: (c)
kept under suspended animation during President's
Exp) Option c is the correct answer.
Rule.
The concept of basic structure was laid down by the Supreme
President's Rule can be imposed for a maximum period
Court in the Keshvananda Bharti Case (1973).
of six months, and its extension beyond six months
requires parliamentary approval every six months. The basic structure doctrine states that the Constitution
of India has certain basic features that cannot be altered or
Currently, Manipur is the only state with highest
destroyed through amendments by the Parliament Article
number (i.e. 10) of President's Rule so far. Uttar
368 does not give absolute powers to the Parliament to
Pradesh has 9 President Rule so far, Bihar and Punjab
amend any part of the Constitution.
both has 8 President Rule as of today.
It can't be exercised by the people through representatives
176. Solution: (c) in a constituent assembly (Statement 2 is incorrect)
Exp) Option c is the correct answer. It falls outside the constituent powers of the parliament.
Under Article-32 of the Constitution of India, individuals (Statement 3 is incorrect)
have the right to move the Supreme Court for the enforcement 179. Solution: (b)
of their Fundamental Rights. However, there are certain
Exp: Option b is the correct answer.
grounds on which the Supreme Court can refuse relief under
Article-32. One such ground is when it is determined that no Under article 21 of the Indian constitution, women's
reproductive choice falls under the right to personal
Fundamental Right has been infringed in the case.
liberty.
177. Solution: (b) The right to personal liberty also includes women's
Exp) Option b is the correct answer reproductive choice the same was held in the case of suchitra
Article 15(2) of the constitution No citizen shall, on srivastava and ors V.chadi. (option 3 is not correct)
grounds only of religion, race, caste, sex, Article 21 of the constitution deals with the protection of life
and personal liberty.
place of birth or any of them, be subject to any disability,
liability, restriction or condition with regard to: No person shall be deprived of his life or personal liberty
according to the procedure established by law. (option 1 is
Access to shops, public restaurants, hotels and palaces of
correct).
public entertainment; or
The use of wells, tanks, bathing Ghats, roads and places 180. Solution: (a)
of public resort maintained wholly or partly out of Exp) Option a is the correct answer.
State funds or dedicated to the use of the general public According to article 28 of the Indian constitution
(option bis correct) Freedom as to attendance at religious instruction or
religious worship in certain educational institutions.
Important Tips:
No religion instruction shall be provided in any
Referred to as the "Magna Carta" of India
educational institution wholly maintained out of State
Fundamental Rights have been mentioned in the Part funds.
III of the constitution from Article 12 to 35.
Nothing in clause ( 1 ) shall apply to an educational
They are justiciable in nature. institution which is administered by the State but has

PYQ W o r k b o o k - - - - - - - - - - - - - - - - - - - - - - - - - - - - - - - - - - - - - l l B I

KING R QUEEN P [ऋषभ राजपूत ]


II INDIAN POLITY AND GOVERNANCE II
in the Concurrent List or State List. This includes the unified framework. Jawaharlal Nehru used this famous
authority to impose taxes not specified in either list. This phrase 'Unity in Diversity' in his book 'Discovery of India'.
provision ensures no legislative gaps and empowers the This phrase is used even today as an expression of harmony
Union Government with residuary powers. and unity amongst different regions and cultures. AU
institutes i.e. Inter-State Councils, National Development
40. Solution: (a)
Council, Finance Commission, Regional Council, Unitary
Exp) Option a is the correct answer Judicial System and All India Services institutes are
The Supreme Court of India holds original jurisdiction, as considered necessary to promote 'Unity among diversity'
per Article 131 of the Indian Constitution, to resolve fiscal in the Indian Federalism.
disputes between the Government of India and one or more
44. Solution: (a)
states, between the Government of India and any state(s)
versus other state(s), and between two or more states. Exp) Option a is the correct answer.
Women, Dalits, Poor and Minority groups are the biggest
41. Solution: (c) stakeholders of Democracy in India. As the constitution
Exp) 0 ption c is the correct answer protects their rights and has many provisions for their
The freedom of trade, commerce, and intercourse is welfare and empowerment such as Fundamental rights,
provided under Part XIII of the Indian Constitution Directive Principle of State Policy (DPSPs), and many
in Articles 301 to 307. Article 301 lays down the general political rights. The weaker section of society needs more
external support and assistance for their empowerment
principles of trade and commerce whereas Article 302 to
unlike the dominant section of society. So maximum
305 enunciates the restrictions which trade is subjected
benefits of a democratic system of governance are taken by
to. Article 306 has been repealed and Article 307 allows
these groups. The reason is that the democracy has emerged
Parliament to appoint authority for implementing Articles
as the carrier for the desire of Self Respect. Their identity
301 to 304. The source for adopting these provisions was the
and dignity are well protected by the constitution of India
Australian Constitution.
and through various parliamentary laws. Hence, both (A)
42. Solution: (b) and (R) are true and (R) is the correct explanation of (A).
Exp) Option b is the correct answer 45. Solution: (c)
Zonal Councils are advisory councils and are made up of Exp) Option c is the correct answer
the states of India that have been grouped into five zones to Assertion (A) is true: Article 356 deals with President's
foster cooperation among them. These were set up vide Part- Rule in case of constitutional breakdown in a state. Sarkaria
III of the States Reorganization Act, 1956. The Union Home Commission recommended using it sparingly, maintaining
Minister is the common chairman of five zonal councils. federal structure, and avoiding undue Central Government
Each chief minister acts as a vice chairman of the council interference in state affairs.
by rotation, holding office for a period of one year at a
Reason (R) is true: The political parties in power at the
time.
Centre have frequently misused Article 356. Since India's
Important Tips independence, it has been invoked over 100 times to impose
President's Rule in various states. In most cases it was used for
The Northeastern states are not covered by any of
political considerations rather than any genuine breakdown
the Zonal Councils and their special problems are
of constitutional machinery in the States.
addressed by another statutory body, the North
Eastern Council at Shillong, created by the North (R) is the correct explanation of (A): The reason behind
.Eastern Council Act, 1971. the Sarkaria Commission's recommendation to use Article
356 sparingly was precisely because it had been misused by
This council originally comprised Arunachal Pradesh,
the political parties that came to form the Government at
Assam, Manipur, Meghalaya, Mizoram, Nagaland
the Centre.
and Tripura; later the state of Sikkim was also added
vide North Eastern Council (Amendment) Act, 2002 46. Solution: (b)
notified on 23 December 2002.
Exp) Option b is the correct answer.
The union territories of Andaman and Nicobar The federal system was adopted in India to safeguard and
Islands and Lakshadweep are not members of any promote the unity of the country, and accommodate regional
of the Zonal Councils. However, they are presently diversity. Such accommodation of diversities has built a
special invitees to the Southern Zonal Council. stronger nation. Most of Indian States have different ethnicity,
culture, languages, etc, but still India as nation stood firm
43. Solution: (d)
and robust. Federal system was designed to facilitate many
Exp) Option d is the correct answer. diverse parts and protect national integration because
The Constitution establishes a federal system of government, accommodation of these diversities has built a stronger
where power is divided between the center and the states. Indian nationhood. Hence, both (A) and (R) are true, and
This allows for regional autonomy and diversity within a (R) is the correct explanation of (A).

1 1 1 - - - - - - - - - - - - - - - - - - - - - - - - - - - - - - - - - - - - - P Y Q Workbook

KING R QUEEN P [ऋषभ राजपूत ]


II INDIAN POLITY AND GOVERNANCE II
47. Solution: (c) Clause (1) empowers the President, with the
Exp) Option c is the correct answer consent of the State Governor, to entrust functions,
The Gadgil-Mukherjee Formula, devised in 1969, allocated conditionally or unconditionally, to the State
central assistance to states based on population, per capita Government or its officers concerning matters falling
income, and special problems. It is no longer in use, and under the executive power of the Union.
allocation now occurs through grant-in-aid, centrally
Clause (2) enables Parliament to make laws that
sponsored schemes, and tax sharing as per Finance
apply to a State, even if the State Legislature lacks
Commission recommendations. The Planning Commission
the authority to enact laws on that subject. It allows
was replaced by NITI Aayog in 2015, leading to changes
the conferment of powers and imposition of duties on
in resource allocation mechanisms in India's fiscal federal
the State or its officers and authorities.
system.
Clause (3) addresses the reimbursement of any
48. Solution: (b)
additional administrative costs incurred by the State
Exp) Option b is the correct answer due to the exercise of powers and duties conferred
Under Article 246 of the Indian Constitution, which deals upon it under this article. The Government of India
with the distribution oflegislative powers between the Union pays an agreed amount, or in case of disagreement,
and the States, "Land Revenue" is exclusively imposed and an arbitrator appointed by the Chief Justice of India
collected by the States under the State List (List II). The determines the compensation.
State Governments have the authority to levy taxes on land
and agricultural income, and they exercise this power as part 52. Solution: (d)
of their revenue generation. Exp) Option dis the correct answer.

49. Solution: (c) A unitary government is one in which all the powers are
vested in the national government. The regional governments,
Exp) Option c is the correct answer
if exist, derive their authority from the national government.
Under Article 270 of the Indian Constitution, taxes on
income, excluding agricultural income, are levied and The Indian Constitution also contains a large number of
collected by the Government of India. These taxes' net unitary or non-federal features such as a strong Centre,
proceeds, except those attributed to Union territories or single Constitution, single citizenship, flexibility of
taxes on Union emoluments, are not part of the Consolidated Constitution, integrated judiciary, appointment of state
Fund of India but are assigned to States where the tax is governor by the Centre, all-India services, emergency
leviable. These proceeds are distributed among the States provisions (Constitutional crisis), President's approval on
as prescribed. state bills, no equality of state representation and so on.
50. Solution: (b) Whereas, some of federal features are, Written Constitution,
Exp) Option b is the correct answer Rigid Constitution, Supremacy of the Constitution,
Decentralization of Powers (7 th Schedule), independent
The Sarkaria Commission recommended establishing
judiciary headed by the Supreme Court (which hears cases
a Permanent Inter-State Council (Inter-Governmental
Council) to foster cooperation and resolve disputes between related center and States), Bicameralism and so on.
the central government and states, promoting federal Federal Government Unitary Government
principles, dialogues, and consensus-building for governance
1. Dual Government (that 1. Single government,
and policy implementation.
is, national government that is) the national
51. Solution: (a) and regional government which
Exp) Option a is the correct answer government) may create regional
governments
Article 257(1) states that, "The executive power of every
State shall be so exercised as not to impede or prejudice 2. Written Constitution 2. Constitution may be
the exercise of the executive power of the Union, and the written (France) or
executive power of the Union shall extend to the giving of unwritten (Britain)
such directions to a State as may appear to the Government 3. Division of powers 3. No division of powers.
of India to be necessary for that purpose". This provision between the national and All powers are vested in
enables the Central Government to intervene when necessary regional government the national government
to maintain the integrity and unity of the nation and to
4. Supremacy of the 4. Constitution may be
ensure the smooth functioning of the federal structure.
Constitution supreme (Japan) or may
Important Tips not be supreme (Britain)
Article 258 of the Constitution of India, deals with the 5. Rigid Constitution 5. Constitution may be
"Power of the Union to confer powers, etc, on States in rigid (France) or flexible
certain cases." (Britain)

PYQ W o r k b o o k - - - - - - - - - - - - - - - - - - - - - - - - - - - - - - - - - - - - - , 1 1 1

KING R QUEEN P [ऋषभ राजपूत ]


II II
6. Independent judiciary 6. Judiciary may be Characteristics features of Indian Federalism:
independent or may not Distribution of Powers: Powers divided between
be independent central and state governments. Schedule 7 divides
Bicameral legislature 7. Legislature may be subjects between Centre and States and maintains an
bicameral (Britain) or extensive concurrent list over which both centre and
unicameral (China) states have jurisdiction. Residuary power rests with
Central Government.
53. Solution: (b)
Dual government: India has a dual polity system with
Exp) Option bis the correct answer separate governments at the central and state levels.
The Northern Zonal Council is a zonal council that comprises Written Constitution: Federal structure defined by
the states and union territories of Chandigarh, National written constitution.
Capital Territory of Delhi, Haryana, Himachal Pradesh, Bicameralism: The Constitution establishes a
Jammu and Kashmir, Punjab, Rajasthan and Ladakh. bicameral legislature comprising the Rajya Sabha and
Important Tip: Lok Sabha.
Five Zonal Councils were set up vide Part-III of the States Supremacy of Constitution: Constitution is supreme
Re-organisation Act, 1956. The present composition of law.
each of these Zonal Councils is as under: Independent Judiciary: Judiciary interprets and
The Northern Zonal Council, comprising the States enforces Constitution.
of Haryana, Himachal Pradesh, Punjab, Rajasthan, Rigidity of Constitution: The Constitution's
National Capital Territory of Delhi and Union Territory rigidity requires joint action of the Central and state
of Chandigarh, Jammu & Kashmir and Ladakh. governments to amend provisions related to the federal
structure.
The Central Zonal Council, comprising the States of
Chhattisgarh, Uttarakhand, Uttar Pradesh and Madhya 55. Solution: (b)
Pradesh;
Exp) Option b is the correct answer
The Eastern Zonal Council, comprising the States of
Bihar, Jbarkhand, Orissa, Sikkim and West Bengal; The M.M. Punchhi Commission on Centre-State Relations
The Western Zonal Council, comprising the States of recommended that a bill reserved for the consideration
Goa, Gujarat, Maharashtra and the Union Territories of the Union Executive should be disposed of within six
of Daman & Diu and Dadra & Nagar Haveli; months. The Punchhi Commission, officially known as the
The Southern Zonal Council, comprising the States of Commission on Centre-State Relations, was established in
Andhra Pradesh, Karnataka, Kerala, Tamil Nadu and
2007 to examine and review the relationship between the
the Union Territory of Puducherry.
central government and state governments in India.
The North Eastern States i.e. (i) Assam (ii) Amnachal
Pradesh (iii) Manipur (iv) Tripura (v) Mizoram (vi)
56. Solution: (b)
Meghafaya and (vii) Nagaland are not included in the
Zonal CmmciJs and their special problems are looked Exp) Option b is the correct answer.
after by the North Eastern Council, set up under the Article-263 in Part XI of the Constitution provides for
North Eastern Council Act, 1972.
the establishment of an lnter"State Counci for better
54. Solution: (a) coordination among the states and between Centre and
Exp) Option a is the correct answer. States.
The Indian Federation is not the result of an agreement In 1990, the Inter-State Council was established on the
among the states, unlike the American Federation.
recommendation of Sarkaria Commission (1983-87)
Article l describes India as a 'Union of States' which implies
The present composition of the Council is as follows:
two things: one, Indian Federation is not the result of an
agreement by the states; and two, no state has the right to (I) Prime Minister as the Chairman
secede from the federation.
(II)Chief Minister of all the States and Union Territories
Important Tips (III) Administrators of Union Territories not having
Indian Federation system is based on the Canadian Assemblies
Model, where a unitary state transforms into a federation
by granting provinces autonomy to promote regional (IV) Governor's of States under President rule
interests. (V) Six Central Cabinet Ministers

mli--------------------------------------PYQWorkbook

KING R QUEEN P [ऋषभ राजपूत ]


II II
Important Tips subjects, such as criminal law, civil law, taxation, labor
Punchhi Commission (2007) : is related to the laws, and more. Statutes are an important source of law in
Commission on Centre-State relations. India.
Raiamannar Commission (1969) : is related to Option 3 is correct- Customary law refers to the practices,
examine all aspects of centre and state. customs, and traditions that are followed by a specific
Mungerilal Commission ( 1977) Identification community or group of people. Customary laws have
backward castes in Bihar.
legal recognition and are applicable in specific regions or
57. Solution: (d) communities, particularly in matters related to personal
laws, inheritance, marriage, and other cultural practices.
Exp) Option d is the correct answer.
Option 4 is correct- Case law, also known as judicial
Indian federalism rather follows the principle of a "blended"
precedent, refers to the decisions and interpretations made
or "cooperative" federalism, where there is a significant by the courts while deciding legal disputes. The judgments
degree of cooperation and coordination between the Union delivered by higher courts, particularly the Supreme Court
Government and the State Governments. The Constitution of India, create precedents that serve as a source of law.
provides for a distribution of powers between the Union These precedents help in interpreting laws and establishing
and the States (through Union and States list), but it also legal principles for future cases.
allows for concurrent powers and shared responsibilities.
59. Solution: (a)
Important Tip Exp) Option a is the correct answer.
Some of the features of Indian Federalism:
A. Pluralist democracy - (4) Capacity of groups to ensure
1. Division of Powers: Clear division of powers between Governmental responsiveness. Pluralist democracy refers
the Union Government and State Governments. to a system where power is dispersed among different
2. Written Constitution: Indian federalism is based on a interest groups, and these groups have the ability to
written Constitution. influence government decisions and policies.
3. Supremacy of the Constitution: The Constitution is B. People's democracy - (3) Social equality through
the supreme law of the land. common ownership of wealth. People's democracy
4. Independent Judiciary: Presence of an independent aims for social equality by advocating for common
judiciary to interpret and protect the Constitution. ownership of wealth and resources, ensuring equitable
distribution among all members of society.
5. Dual Polity: Recognition of a dual polity consisting of
the Union Government and State Governments. C. Developmental democracy (2) Highest and
harmonious development of individual capabilities.
6. Rigid Constitution: The Indian Constitution requires Developmental democracy focuses on promoting the
special procedures for amendments. holistic development of individual capabilities,
7. Distribution of Powers: Powers allocated through allowing individuals to reach their full potential in a
Union List, State List, and Concurrent List. harmonious society.
8. Strong Center: The Union Government holds more D. Elitist democracy - (1) Power is always exercised by a
extensive powers compared to State Governments. privileged few. Elitist democracy refers to a system where
9. Cooperative Federalism: Emphasis on collaboration power is concentrated in the hands of a small, privileged
and coordination between Union and State group, often excluding the majority of the population
Governments. from decision-making processes.
10. Independent Election Commission: An independent 60. Solution: (a)
body responsible for conducting elections at national Exp) Option a is the correct answer.
and state levels.
In a presidential form of government, the President is
58. Solution: (d) separate from the legislative body, which means that the
President is not a member of the legislature. The President is
Exp) Option d is the correct answer.
the head of the executive branch and is elected independently
Option I is correct- The Constitution of India is the of the legislature. This separation of powers ensures a clear
supreme law of the land. It lays down the fundamental distinction between the executive and legislative functions.
principles, rights, and duties of citizens, and establishes Option b is incorrect: The Presidential form of government
the structure and functioning of the government. The does separate the legislative and executive functions,
Constitution serves as the foundation and source of all laws with the President being the head of the executive branch
in India. and the legislature having its own separate powers and
Option 2 is correct- Statutes, also known as legislation or responsibilities.
acts, are laws enacted by the Parliament of India or the Option c is incorrect: The principle of collective
State Legislative Assemblies. These laws are formulated and responsibility is usually associated with parliamentary
passed by the respective legislative bodies and cover various systems, where the executive (Prime Minister and

PYQ W o r k b o o k - - - - - - - - - - - - - - - - - - - - - - - - - - - - - - - - - - ~ 1 1 1 1
i)Forum~

KING R QUEEN P [ऋषभ राजपूत ]


II INDIAN POUTY AND .GOVERNANCE II
Cabinet) is collectively responsible to the legislature. In 63. Solution: (c)
a presidential system, the President is not bound by the Exp) Option c is the correct answer.
principle of collective responsibility.
Statement 1 is correct: Residuary powers have been given
Option dis incorrect: In a presidential form of government, to the Union Parliament by Article 248 and Entry 97 of the
the tenure of the President is typically fixed and does not Union List. These provisions give Parliament exclusive power
depend on the legislature. The President serves a fixed term to make any law with respect to any matter not enumerated
as specified by the constitution and is not subject to removal in List II or List III, including any tax not mentioned in
by the legislature, except under specific circumstances either of those lists.
outlined in the constitution. Statement 2 is incorrect: In the matter of residuary powers,
the Constitution of India does not follow the Constitution
61. Solution: (c)
of Australia, but follows the Constitution of Canada. The
Exp) Option c is the correct answer. Constitution of Australia does not vest the residuary powers
Statement a is correct: Indian federalism grants powers to in any level of government, but leaves them to be determined
both the central government and the state governments. by judicial interpretation. The Constitution of Canada, on
Each state government has its own legislative and executive the other hand, vests the residuary powers in the federal
powers to govern their respective territories. government by Section 91 of its Constitution Act, 1867.
Statement b is correct: The judiciary in India, particularly Statement 3 is incorrect: Schedule 7 of the Constitution
the Supreme Court, has the authority to interpret the of India does not provide a list of residuary powers, but
constitution and determine the scope of powers and provides three lists of subjects on which Parliament and state
jurisdiction of different levels of government. The judiciary legislatures can make laws. These are List I (Union List), List
acts as a neutral arbiter in resolving disputes between the II (State List) and List III (Concurrent List). The residuary
central government and state governments. powers are those that arc not covered by any of these lists.

Statement c is incorrect: States are subordinate to the Statement 4 is correct: Government oflndia Act, 1935 placed
Central Government: This statement is not a feature of residuary powers in the hands of the Governor-General.
This was done to ensure that no important subject was left
Indian federalism. In Indian federalism, both the central
out from legislative competence and to give flexibility to deal
government and state governments are sovereign in their
with unforeseen situations. The Governor-General could
respective spheres of power. While the central government
legislate on any matter that was not included in any of the
has certain powers and responsibilities that are applicable
seven lists provided by this Act
to the entire country, state governments also possess
independent powers to govern their respective states. 64. Solution: (c)
Statement d is correct: Indian federalism entails a division Exp) Option c is the correct answer.
of fiscal powers between the central government and Statement l is correct: The head of government is also
state governments. While the Constitution provides for the the head of state in the presidential system. The president
assignment of revenue sources to each level of government, represents the nation both internally and externally, and has
the distribution of fiscal resources can be subject to periodic the authority to appoint and dismiss cabinet members, issue
revisions and negotiations between the central government executive orders, veto legislative acts, negotiate treaties,
and state governments. command the armed forces, etc.

62. Solution: (b) Statement 2 is correct: Executive can veto legislative acts
in the presidential system. The president has the power to
Exp) Option b is the correct answer.
reject or approve bills passed by the legislature, and can also
The setting up of the Inter-State Council in 1990 was propose legislation to the legislature. However, the legislature
aimed at strengthening the federal provisions of the can override the president's veto by a supermajority vote in
Constitution of India. The Inter-State Council serves as some cases
a platform for coordination and cooperation between the
Union government and the state governments. It facilitates 65. Solution: (a)
the discussion and resolution of issues and disputes between Exp) Option a is the correct answer
the Union and states, promoting cooperative federalism. Article 300 of the Indian Constitution deals with "Suits
and proceedings." It states that the Union Government
Important Tips
(Government oflndia) and State Governments (Government
The Inter-State Council is a non-permanent of a State) can sue or be sued in their respective names.
constitutional body that was established by a
presidential order on 28 May 1990 based on Article 66. Solution: (c)
263 of the Constitution of India. Exp) Option c is the correct answer.
The main purpose of the Inter-State Council is to The Democracy of India is based on the fact that public
enhance the coordination between the Centre and the enjoys the rights to choose and change the Government.
States and to discuss or investigate policies, subjects of Democracy refers to a system of government where the
common interest, and disputes among States citizens exercise power by voting.
-··· -··-······· ---------~

I J a f - - - - - - - - - - - - - - - - - - - - - - - - - - - - - - - - - - - - - - - P Y Q Workbook

KING R QUEEN P [ऋषभ राजपूत ]


II
Important Tips
Key characteristics of Democracy:
1. Democracy is of two types-direct and indirect.
2. In direct democracy, the people exercise their supreme power directly as is the case in Switzerland. There are four devices
of direct democracy, namely, Referendum, Initiative, Recall and Plebiscite.
3. In indirect democracy, on the other hand, the representatives elected by the people exercise the supreme power and thus
carry on the government and make the laws.
This type of democracy, also known as representative democracy, is of two kinds-parliamentary and presidential.
The Indian Constitution provides for representative parliamentary democracy under which the executive is responsible
to the legislature for all its policies and actions.
4. The term 'democratic' is used in the Indian Preamble in the broader sense embracing not only political democracy but
also social and economic democracy. This dimension was stressed by Dr. Ambedkar in his concluding speech in the
Constituent Assembly on November 25, 1949, in the following way: "Political democracy cannot last unless there lies at
the base of it social democracy.

•••

PYQ W o r k b o o k - - - - - - - - - - - - - - - - - - - - - - - - - - - - - - - - - - - - - 1 1 1
mForun,~

KING R QUEEN P [ऋषभ राजपूत ]


II II
Important Tips However, the Kesavananda Bharati case later
Lok Adalats have been given statutory status under overturned the Golak Nath judgment, stating that
the Legal Services Authorities Act, 1987. while Parliament can amend the Constitution, it
cannot destroy its basic structure.
The Lok Adalat have the same powers as are vested
in a Civil Court under the Code of Civil Procedure 66. Solution: (b)
(1908). Exp) Option b is the correct answer
Lok Adalat have the requisite powers to specify its own The case of Indra Sawhney Vs Union of India, commonly
procedure for the determination of any dispute coming known as the Mandal Commission case, dealt with the issue
before it. of the "creamy layer" among the backward classes in India.
Under the said Act, the award (decision) made by the Important Tip:
Lok Adalats is deemed to be a decree of a civil court
The Supreme Court, in its judgment, upheld the
and is final and binding on all parties and no appeal
validity of reservations for OBCs but introduced the
against such an award lies before any court of law.
concept of the "creamy layer:'
64. Solution: (b) It defined the "creamy layer" as the relatively better-
Exp) Option b is the correct answer off and more privileged individuals within the OBC
In the SP Gupta vs. President of India case, which is also category who should be excluded from the benefits of
reservations.
known as the First Judges' Case, the Supreme Court of
India held that the "primacy" of the CJl's (Chief Justice This exclusion was based on the rationale that
of India) recommendation on judicial appointments and reservations should reach those who genuinely
transfers can be refused for "cogent reasons." The ruling need them and uplift the most socio-economically
gave the Executive primacy over the Judiciary in judicial disadvantaged sections.
appointments for the next 12 years. KM Nanavati v. State of Bombay was a case about the
right to equality, Madhu Limaye v. Ved Murti was a
Important Tips
case about the right to freedom of speech, and Sajjan
I. Second Judges Case (1993): Singh v. State of Punjab was a case about the right to
SC introduced the Collegium system, holding that property.
"consultation" really meant "concurrence".
67. Solution: (c)
It added that it was not the CJI's individual opinion,
Exp) Option c is the correct answer.
but an institutional opinion formed in consultation
with the two senior-most judges in the SC. The locus standi rule to move the court was liberalized by
the case of SP Gupta vs Union of India. This case is also
2. Third Judges Case (1998): known as the Judges' Transfer Case, as it dealt with the issue
SC on the President's reference (Article 143) expanded of transfer of judges from one High Court to another by the
the Collegium to a five-member body, comprising the President in consultation with the Chief Justice of India.
CJI and four of his senior-most colleagues.
Important Tip
65. Solution: (b) SR Bommai vs Union of India was a case that dealt
Exp) Option bis the correct answer with the constitutional validity of President's rule
imposed under Article 356 in various states. The
The Golak Nath case was a landmark judgment of the
court laid down certain safeguards and limitations on
Supreme Court of India in 1967 that dealt with the issue
the exercise of this power by the Centre and upheld the
of the power of Parliament to amend the Constitution. federal strncture of the Constitution.
The Court held that Parliament could not amend any of the
Minerva Mills vs Union of India was a case that
provisions of Part III of the Constitution, which contains the
dealt with the constitutional validity of certain
Fundamental Rights. The Court held that these rights were amendments made to Article 31C and Article 368 by
fundamental and could not be taken away by Parliament. the 42nd Amendment Act, 1976. The court struck down
Important Tip these amendments as violative of the basic strncture
doctrine and upheld the supremacy of judicial review
In the Golak Nath case, the Supreme Court declared
and fundamental rights over directive principles.
the 24th Amendment, which amended Article 31
regarding the right to property, as unconstitutional. Kesavananda Bharati vs State of Kerala was a case
that dealt with the constitutional validity of certain
The Court held that an amendment under Article amendments made to Article 31 by various land
368 is "law" within the meaning of Article 13 of the reform acts. The court propounded the basic structure
Constitution and therefore, if an amendment "takes doctrine and held that Parliament had 110 power to
away or abridges" a Fundamental Right conferred by amend any part of the Constitution that formed its
Part III, it is void. essential core or identity .

PYQ W o r k b o o k - - - - - - - - - - - - - - - - - - - - - - - - - - - - - - - - - - - - 1 1 1 1
mForum!m

KING R QUEEN P [ऋषभ राजपूत ]


II INDIAN POLITY AND GOVERNANCE II
68. Solution: (c) 71. Solution: (a)
Exp) Option c is the correct answer Exp) Option a is the correct answer.
Statement 1 is correct: The condition mentioned in statement The writ of Habeas Corpus is a legal remedy that allows
1 implies that the writ of certiorari can only be issued against an individual to seek relief from unlawful detention or
a tribunal or an officer who has the legal authority to decide imprisonment. The term "Habeas Corpus" is derived from
matters that impact the rights of individuals and is obligated
Latin, which means "you may have the body."
to act judicially. In other words, the tribunal or officer must
have the power to make decisions that affect the rights of Important Tip
individuals, and they must have a duty to act in a judicial or
l. Article 32 of the Indian Constitution provides for
quasi-judicial capacity.
the right to constitutional remedies to protect
Statement 2 is correct: This condition implies that the writ
Fundamental Rights and it is integral to Basic Structure
of certiorari can only be issued if the tribunal or officer has
doctrine.
acted beyond their legal authority or jurisdiction. If they
have acted without the requisite jurisdiction, the writ can be 2. Five Constitutional Writs:
issued to quash their decision or order. Habeas Corpus: It used to ensure the release of a
For the issue of a writ of certiorari in India, both conditions person who is unlawfully detained or imprisoned
mentioned in options 1 and 2 are necessary. Mandamus: It issued to compel public officials or
Important Tip authorities to perform their duties or functions.
Concept of Certiorari Prohibition: It is used to prohibit lower courts or
It literally means 'to be certified' or 'to be informed: tribunals from exceeding their jurisdiction or acting
It is issued by a higher court to a lower court or contrary to law.
tribunal to either transfer a case pending with the Certiorari: It is issued to quash the orders or
latter or to overturn the latter's order in a case.
decisions of lower courts or tribunals on grounds of
It is issued due to an excess of jurisdiction, a lack of jurisdictional errors or violation of natural justice.
jurisdiction, or a legal error.
Quo Warranto: It is used to inquire into the legality of
As a result, unlike prohibition, which is only
preventative, certiorari is both preventive and a person holding a public office and to challenge their
curative. authority or credentials.
Previously, the writ of certiorari could only be issued
72. Solution: (a)
against _\•::l.'dal and quasi-judicial authorities, not
administrative ones. Exp) Option a is the correct answer
Howev?r, th-c S 1preme Court ruled in 1991 that The power to establish a common High Court is vested in
certiorari ;:,.,. :•" ':sued even against administrative the Parliament, under Article 231 of the Constitution.
authoritie,i ,:Jfr.:l.int individual rights.
Certiorari, like prohibition, is not available against 73. Solution: (c)
legislative bo<l:-~s or;: ~•vate individuals or bodies. Exp) Option c is the correct answer.
69. Solution: (a) Under Article-32 of the Constitution of India, individuals
Exp) Option a is the correct answer have the right to move the Supreme Court for the enforcement
In December 2014, when Justice HL Dattu was the Chief of their Fundamental Rights. However, there are certain
Justice of India (CJI), the Supreme Court had issued a grounds on which the Supreme Court can refuse relief under
notification to set up a special Bench known as the Social Article-32. One such ground is when it is determined that no
Justice Bench. The Social Justice Bench was established to Fundamental Right has been infringed in the case.
exclusively hear cases related to social justice issues. It aimed
to expedite the hearing and disposal of cases pertaining to 74. Solution: (a)
matters such as gender inequality, child rights, disability Exp) Option a is the correct answer.
rights, environmental protection, and other socio-economic Under Article 131 of the Constitution of India, the Supreme
concerns. Court has original jurisdiction in matters related to disputes
70. Solution: (d) between states, or between the Government of India and one
Exp) Option d is the correct answer. or more states, or between two or more states on one side
and the Government of India on the other.
The case of "Naz Foundation vs. Government of NCT of
Delhi and others" addressed the constitutionality of Section Important Tip
377 of the Indian Penal Code, which criminalized same-
1. The Supreme Court has exclusive original jurisdiction
sex consensual activities. The court's decision impacted
in federal disputes, which include disputes between
fundamental rights under Articles 21, 14, and 15 of the
the Centre and one or more states, or between
Indian Constitution, including the right to life, equality, and
different states.
prohibition of discrimination.

l l l f - - - - - - - - - - - - - - - - - - - - - - - - - - - - - - - - - - - - - - P Y Q Workbook

KING R QUEEN P [ऋषभ राजपूत ]


II INDIAN POLITYAND GOVERNANCE II
into the authority by which the person claims to hold the
2. This jurisdiction is limited to disputes involving
questions of law or fact that determine the existence office and whether they have the legal right to do so. The
or extent of a legal right, excluding political questions. reason option (c) is not a necessary condition for issuing
a writ of quo warranto is that the eligibility or legality
3. The Supreme Court cannot entertain suits brought
by private citizens against the Centre or a state under of holding a substantive office is equally important to
this jurisdiction. ensure that the person holding the office has the necessary
qualifications and meets the requirements laid out by law
4. The Supreme Court's jurisdiction does not extend
to disputes arising from pre-Constitution treaties, or the constitution.
agreements, or certain matters such as inter-state water Important Tip
disputes, matters referred to the Finance Commission,
1. 'Quo Warranto' means 'by what warrant'. Through
commercial disputes, or recovery of damages by a state
this writ, the Court calls upon a person holding a public
against the Centre.
office to show under what authority he holds that
5. In 1961, the Supreme Court dismissed the first suit office. If it is found that the person is not entitled to
brought under its original jurisdiction, where West hold that office, he may be ousted from it. Its objective
Bengal challenged the Constitutional validity of the is to prevent a person from holding an office he is
Coal Bearing Areas (Acquisition and Development) not entitled to, therefore preventing usurpation of
Act, 1957, passed by the Parliament. any public office. It cannot be issued with respect to a
private office.
75. Solution: (c)
2. The writ can be issued only when the following
Exp) Option c is the correct answer
conditions are fulfilled:
Both the Supreme Court and High Courts have the
authority to issue writs. While the Supreme Court primarily The public office is wrongfully assumed by the
private person.
issues writs for the enforcement of fundamental rights, High
Courts can issue writs not only for fundamental rights but The office was created by the constitution or law and
also for other purposes, such as the enforcement of legal the person holding the office is not qualified to hold
rights or statutory duties. the office under the constitution or law.

Important Tip The term of the public office must be of a permanent


nature.
Article 32, Constitution of India 1950
1. The right to move the Supreme Court by appropriate The nature of duties arising from the office must be
public.
proceedings for the enforcement of the rights conferred
by Part III is guaranteed. 77. Solution: (c)
2. The Supreme Court shall have power to issue directions
Exp) Option c is the correct answer.
or orders or writs, including writs in the nature of
habeas corpus, mandamus, prohibition, quo warranto The concept of basic structure was laid down by the
and certiorari, whichever may be appropriate, for the Supreme Court in the Keshvananda Bharti Case (1973).
enforcement of any of the rights conferred by this Part. The basic structure doctrine states that the Constitution
3. Without prejudice to the powers conferred on the of India has certain basic features that cannot be altered
Supreme Court by clauses (I) and (2), Parliament or destroyed through amendments by the Parliament
may by law empower any other court to exercise Article 368 does not give absolute powers to the
within the local limits of its jurisdiction all or any of Parliament to amend any part of the Constitution.
the powers exercisable by the Supreme Court under
It can't be exercised by the people through representatives
clause (2).
The right guaranteed by this article shall not be suspended in a constituent assembly (Statement 2 is incorrect)
except as otherwise provided for by this Constitution. It falls outside the constituent powers of the parliament.
Similarity, Under Art 226, Constitution of India - High (Statement 3 is incorrect)
Court have the power to issue writes.
The write jurisdiction of High Court is wider than that 78. Solution: (a)
of Supreme Court , as High Courts can issue writes with
respect to Fundamental as well as legal Rights , whereas Exp) Option a is the correct.
the supreme court can issue writes with respect to only In Bachan Singh Vs State of Punjab (1980), Supreme
fundamental rights.
Court laid down that Capital Punishment in India would
76. Solution: (c) be given only in the rarest of rare cases.
Exp) Option c is the correct answer It stated that the principle that the death penalty ought not
A writ of Quo Warranto is a legal action that challenges to be awarded unless the alternative of life imprisonment
a person's right to hold a public office. It seeks to inquire is "unquestionably foreclosed".

PYQ W o r k b o o k - - - - - - - - - - - - - - - - - - - - - - - - - - - - - - - - - - - - - - - - - , 1 1 1
fa FonimlrA'i

KING R QUEEN P [ऋषभ राजपूत ]


II II
Important Tip 82. Solution: (b)
Gopalanachari Vs State of Kera la (1980) : Person shall Exp) Option b is the correct answer.
be given legal aid at the expense of the state. Golak Nath case held that constitutional amendment
Dr. Upendra Baxi Vs State of Uttar Pradesh (1983) : passed according to article 368 is a law within the
seeking enforcement of the constitutional rights. meaning of article 13.
Tukaram Vs State of Maharashtra (1979) : There is a 24th amendment act 1971 override this judgement and
word of difference between sexual intercourse & rape. declare that amendment under article 368 is not law
79. Solution: (b) within the meaning of article 13.

Exp) Option b is the correct answer Important Tip


Daniel Latifi Vs Union of India: In this case a divorced Keshavanand Bharati Case is related to the basic
Muslim women is entitled to reasonable and fair structure doctrine.
provision and maintenance from her former husband and Minerva Mills Case is related to the interpretation of
this should be paid within the period of iddat. the basic structure doctrine.
Muhammad Ahmed Khan Vs Shah Bano Begum : In Maneka Gandhi Case is related to the Right of Personal
this case Supreme Court stated that divorced women had Liberty granted by Article 21 of the Constitution.
the right to maintenance even after the period of Iddat
was over. 83. Solution: (a)
Mary Roy Vs State of Kerala : landmark case brought Exp) Option a is the correct answer.
equal rights for Syrian Christian women in India as Assertion is true- In the district, the district judge holds
their male siblings on matters of inheritance. the highest judicial authority, having both original and
Shankari Prasad Vs. Union of India : In this case appellate jurisdiction over both civil and criminal matters.
Constitutional validity of the first amendment act( 1951) Part VI (Articles 233 to 237) of the Indian Constitution
was challenged. deals with the District Courts.
80. Solution: (a) Reason (R) is true- The district judge exercises both
Exp) Option a is the correct answer. judicial and administrative powers, and they also possess
supervisory authority over all subordinate courts within the
• The basic structure doctrine was given in the landmark
decision of Kesavananda Bharati v State of Kerala ( 1973 ). district. The High Court hears appeals against the directives
and judgments of the district judge.
The basic structure doctrine states that the Constitution
of India has certain basic features that cannot be altered 84. Solution: (b)
or destroyed through amendments by The Parliament.
Exp) Option b is the correct answer
The Supreme Court can undertake judicial review for
Judicial Review is governed by the principle of "Procedure
such amendments.
established by law" Judicial review is the power of the
The Supreme Court declared that Article 368 did judiciary to examine the constitutionality of executive orders
not enable Parliament to alter the basic structure or
and legislative enactments of both the State and Central
framework of the Constitution and Parliament could not
governments. If, after this exarnination, they arc found to
use its amending powers under Article 368 alter the basic
be violative of the Constitution, they can be declared as
structure of the Constitution. Thus, statement 3 is correct.
ln the Golaknath case, the supreme court rnled that unconstitutional, illegal, and involid (null and void) by the
Parliament couldn't curtail any of the Fundamental judiciary.
rights. (this case is not related to basic structure of the
85. Solution: (d)
constitution). Thus, statement 4 is incorrect
Exp) Option dis the correct answer
81. Solution: (d)
A Judge of the Supreme Court can be removed by
Exp) Option dis the correct answer. the President of India after an hripeachment by the
The Supreme Court is the guarantor and defender of Parliament. As per Article 124(4), A Judge of the Supreme
fundamental rights of the citizen . Court can be removed from office by the President of India
In this regard, Supreme Court has original jurisdiction in only after an address by each H.ouse of Parliament supported
the sense that an aggrieved citizen can directly go to the by a majority of the total membership of that House and by a
Supreme Court, not necessarily by way of appeal. majority of not less than two••thirds of members present and
However, the writ jurisdiction of Supreme Court is not voting, and presented to the President in the same session
exclusive because High Court also empowered of issue for such removal on the ground of proved misbehaviour or
writ jurisdiction (Hence option d is incorrect) incapacity.

1 1 1 1 - - - - - - - - - - - - - - - - - - - - - - - - - - - - - - - - - - - - PYQ Workbook
!11Fon11u(IW

KING R QUEEN P [ऋषभ राजपूत ]


II INDIAN POLITY AND GOVERNANCE II
Important Tips Alternatively, must have been an advocate of a High
Article 124(4) and the Judges Inquiry Act 1968 determine Court for at least ten years or two or more such
the procedure of removal of the judges: courts in succession.
A motion of impeachment addressed to the President is Must be a distinguished jurist in the opinion of the
to be signed by at least 100 members of the Lok Sabha president.
or 50 members of the Rajya Sabha and then delivered
to the Speaker of Lok Sabha or the Chairman of Rajya 89. Solution: (d)
Sabha.
Exp) Option d is the cone ct answer
The motion is to be investigated by a Committee of
3 judges of the Supreme Court and a distinguished Vishaka & ors. v/s state of Rajasthan is a case which deals
jurist. with the evil of Sexual Harassment of a women at her
If the Committee finds the judge guilty of misbehavior workplace. It is a landmark judgment case in the history of
or that he suffers from incapacity, the motion along sexual harassment which as being decide by Supreme Court.
with the report of the committee is taken up for
consideration in the House where motion was moved. Important Tips
The judge is then removed by the requisite majority, In 1997, the Supreme Court in India formulated the
i.e. majority of total and 2/3 of its members present Vishakha guidelines to protect women from sexual
and voting. harassment at workplaces.
The Supreme court made it mandatory for all institutions
86. Solution: (a)
to follow the Vishaka guidelines across the country.
Exp) Option a is the correct answer
The guidelines suggested that the compliance mechanism
Option (i) is True: As per Article 127(1), if there's no should ensure the timebound treatment of complaints.
quorum of Judges available for a Supreme Court session,
The Vishakha guidelines gave rise to the foundation to
the Chief Justice of India, with the President's prior consent
redress the Sexual Harassment of Women at Workplace
and consultation with the relevant High Court's Chief Justice,
(Prevention, Prohibition, and Redressal) Act, 2013.
may request a qualified High Court Judge to serve as an ad
hoc Judge for the required period. 90. Solution: (d)
Option (ii) is False: As per Article 224(1), if there's a
Exp) Option d is the cone ct answer
temporary increase in a High Court's business or anears
of work, the President may appoint qualified persons as Bella Banerjee case (1953) dealt with the constitutionality
additional Judges for up to two years. However, additional of the West Bengal Land Development and Planning Act,
judges are not appointed in the Supreme Court; they are 1948. The Supreme Court held certain provisions of the Act
appointed in High Courts. unconstitutional and void. The judgment highlighted the
87. Solution: (b) importance of providing fair compensation to landowners
Exp) Option b is the correct answer during the process of land acquisition for public purposes
According to Article 139A of the Constitution of India the and established the principle that the government's
Supreme Court has the right to transfer any case anywhere in declaration of public purpose should not be regarded as
the territory of India. The Supreme Court can also withdraw conclusive, leaving room for judicial review. It did not
the case or cases pending before the High Court and dispose concern the right to travel abroad as part of personal liberty.
of all the cases to itself.
Important Tips
88. Solution: (b)
The Indira Sawhney case, also known as the Mandal
Exp) Option b is the correct answer Commission case, 1992 where the Supreme Court
According to Article 124(3) of the Constitution of India, upheld the implementation of reservations for Other
a person can be appointed as a Judge of the Supreme Court Backward Classes (OBCs) in educational institutions
if he/she has been an advocate of a High Court or of two or and public employment. However, the Court ruled
more such Courts in succession for at least 10 years. that the "creamy layer" within OB Cs should be excluded
Important Tips from reservation benefits to ensure that the benefits
reach the truly disadvantaged sections.
According to Article 124(3) of the Constitution, a person
In the Vishakha case of 1997, the Supreme Court
can be appointed as a judge of the Supreme Court if he or
laid down guidelines to prevent sexual harassment
she:
of women at workplaces. These guidelines were later
• A person must be a citizen of India. incorporated into the Sexual Harassment of Women
• Must have served as a judge of a High Court for at at Workplace (Prevention, Prohibition, and Redressal)
least five years or two such courts in succession. Act, 2013.

PYQ W o r k b o o k - - - - - - - - - - - - - - - - - - - - - - - - - - - - - - - - - - - 1 1 1
!mFoturn~

KING R QUEEN P [ऋषभ राजपूत ]


II INDIAN POLITY AND GOVERNANCE II
In the Maneka Gandhi case (1978), the Supreme Court 92. Solution: (d)
held that the right to life and personal liberty (Article Exp) Option d is the correct answer
21) in the Indian Constitution is not limited to mere A Public Interest Litigation can be filed in the High Court
animal existence but includes a meaningful life with or Supreme Court. One can file a PIL under Article 226 of
dignity. The Court emphasized that Articles 14, 19, and the Constitution before the High Court and under Article 32
21 are not mutually exclusive and are interconnected to before the Supreme Court of India.
ensure fundamental rights' comprehensive protection.
Important Tips
91. Solution: (b) Public interest Litigation (PIL) means litigation filed in a
court of law, for the protection of "Public Interest", such as
Exp) Option bis the correct answer
Pollution, Terrorism, Road safety, Constructional hazards
The appellate jurisdiction of the Supreme Court can etc.
be invoked by a certificate granted by the High Court Any matter where the interest of public at large is affected
concerned under Article 132(1), 133(1) or 134 of the can be redressed by filing a Public Interest Litigation in a
Constitution in respect of any judgement, decree or final court of law.
order of a High Court in both civil and criminal cases, Public interest litigation is not defined in any statute or in
involving substantial questions oflaw as to the interpretation any act. It has been interpreted by judges to consider the
of the Constitution. intent of public at large.

•••

1 1 1 - - - - - - - - - - - - - - - - - - - - - - - - - - - - - - - - - - - - - P Y Q Workbook

KING R QUEEN P [ऋषभ राजपूत ]


II II
114. Solution: (c) Delhi and Puducherry (excluding Jammu and Kashmir, as
Exp) Option c is the correct answer specified in the 70th Constitutional Amendment Act).
The Estimates Committee is a standing committee of the 117. Solution: (a)
Lok Sabha, the lower house of the Parliament of India. It Exp) Option a is the correct answer.
is responsible for scrutinizing the government's budgetary
Article 78 of the Indian Constitution pertains to the duties
proposals and suggesting ways to improve their efficiency
of the Prime Minister. It states that it shall be the duty of the
and economy. The committee has 30 members, all of whom
Prime Minister to communicate to the President all decisions
are elected members of the Lok Sabha.
of the Council of Ministers relating to the administration
115. Solution: (a) of the Union and proposals for legislation. Additionally,
the Prime Minister shall furnish such information to the
Exp) Option a is the correct answer
President as the President may call for. Article 78 outlines
According to Article 59 of the Constitution of India, the responsibilities of the Prime Minister in keeping the
The President shall not be a member of either House of President informed about the decisions and functioning of
Parliament or of a House of the Legislature of any State, and the Council of Ministers.
if a member of either House of Parliament or of a House of
the Legislature of any State be elected President, he shall 118. Solution: (c)
be deemed to have vacated his seat in that House on the Exp) Option c is the correct answer.
date on which he enters upon his office as President. Thus The 91st Constitutional Amendment Act, of 2003
any Indian citizen who is eligible to become a member of introduced provisions to address the size of the Council
Parliament can also be elected as the President, irrespective of Ministers, prevent defections, and strengthen the anti-
of whether they are a Union Minister or not. There is no defection law. One of the provisions states that the total
explicit restriction on Union Ministers from being eligible number of ministers, including the Prime Minister, in the
for the position of the President. Central Council of Ministers, should not exceed 15% of the
Important Tip: total strength of the Lok Sabha.
Facts regarding Election of President of India 119. Solution: (c)
The Indian President is elected through an electoral Exp) Option c is the correct answer.
college system, by proportional representation by a As per Article 55(3) of the Constitution of India, the
single transferable vote. The Elections are conducted election of the President shall be held in accordance with
and overseen by the Election Commission of India. the system of proportional representation by means of
The electoral college is based on all elected members single transferable vote and the voting at such election shall
of Parliament (MPs), and the elected members of be by secret ballot. This means that the voting process for
State Legislative Assemblies and Union Territories the election of the President involves the use of the single
(MLAs). transferable vote system, which allows voters to rank the
Before voting, in the nomination stage, the candidate candidates in order of preference.
files nomination with a signed list of 50 proposers and 120. Solution: (a)
50 seconders. These can be anyone from the members of
Exp) Option a is the correct answer
the electoral college from the State or national level. This
rule was implemented by EC in 1974. An elector cannot The National Disaster Management Authority (NDMA) is
propose or second the nomination of more than one the apex statutory body for disaster management in India
candidate. and is headed by the Prime Minister. As the head of the
NDMA, the Prime Minister oversees the overall functioning
A vote cast by each MP or MLA is not calculated as
of the organization and provides strategic direction for
one vote.
disaster management initiatives.
The fixed value of each vote by an MP of the Rajya
Sabha and the Lok Sabha is 700. Important Tip
Meanwhile, the vote value of each MLA differs from Mandate: Its primary purpose is to coordinate response to
State to State based on a calculation that factors in natural or man-made disasters and for capacity-building
its population vis-a-vis the number of members in its in disaster resiliency and crisis response. It is also the
legislative Assembly. apex body to lay down policies, plans and guidelines
for Disaster Management to ensure timely and effective
116. Solution: (c) response to disasters.
Exp) Option c is the correct answer. Vision: To build a safer and disaster resilient India by
The President of India is elected indirectly by an electoral a holistic, proactive, technology driven and sustainable
college composed of the elected representatives (and not all development strategy that involves all stakeholders
members) from the Parliament of India and the Legislative and fosters a culture of prevention, preparedness and
Assemblies of the States, as well as the Union Territories of mitigation.

PYQ W o r k b o o k - - - - - - - - - - - - - - - - - - - - - - - - - - - - - - - - - - - - - - 1 1 1

KING R QUEEN P [ऋषभ राजपूत ]


II INDIAN POUTYANDGOVI~RNANCE II
Divisions: NDMA-h~S-;~;ajor divisions viz. Policy & But under the 7th Amendment Act, 1956, the same
Plans, Mitigation, Operations & Communications person can be appointed as Governor of one or more
& Information & Technology, Administration and States.
[
Finance. Following are the powers and duties performed by
····- ·-·-----~--- - ---
Governor in states, they are as follows:
121. Solution: (d)
(i) They have power to appoint Advocate General,
Exp) Option dis the correct answer.
(ii) They can summoning, proroguing and dissolving State
Article 75 of the Constitution states that the Prime Minister
Legislature,
has the authority to appoint ministers, including those who
(iii) They have power to grant pardons reprieves, respites
are not members of Parliament. However, non-member
or remission of punishments.
ministers are required to become members of either the
Lok Sabha or the Rajya Sabha within 6 months in order to 126. Solution: (d)
continue serving as ministers. Exp) : Option dis the correct answer

122. Solution: (a) Article 213 empowers the Governor to promulgate


Ordinance, during recess of legislature.
Exp) Option a is the correct answer
The Vice-President of India is eligible for re-election. Important Tip
There is no restriction on the number of terms a Vice- Governor of an Indian state draws ordinance making
President can serve. The Vice-President can be re-elected power from Article 213 of the Constitution.
for multiple terms if the person is chosen by the Electoral The Governor can only issue ordinances when the
College again and meets the eligibility criteria. Legislative Assembly of a state or where there are two
houses in a state both houses are not in session.
123. Solution: (c)
Governor's ordinance-making power is not a
Exp) Option c is the correct answer discretionary power.
The President's resignation is governed by Article 56 of the This means that he can promulgate or withdraw an
Indian Constitution. According to this article, the President ordinance only on the advice of the council of ministers
can resign by writing a letter to the Vice President. headed by the chief minister.
··----------------'

124. Solution: (c) 127. Solution: (d)


Exp) Option c is the correct answer Exp) Option d is the correct answer
Article 176 states that at the commencement of the first Ordinance making power of the governor is not a
session after each general election to the Legislative Assembly discretionary power, rather it comes under legislative
and at the commencement of the first session of each year, power of a governor (option l is incorrect)
the Governor shall address the Legislative Assembly or, in The governor is himself competent to withdraw the
the case of a State having a Legislative Council, both Houses ordinance at any time. (option 2 is incorrect)
assembled together. This is also called as special address. 128. Solution: (h)
125. Solution: (a) Exp) Option b is the con-ect answer.
Exp) Option a is the correct answer The duties of the Governor as a Constitutional Head of
the State do not become the subject matter of questions
A governor possesses executive, legislative, financial, and
or debate in the Parliament, is not correct , As any
judicial powers
matter of utmost importance can be discussed in the
However, he has no diplomatic, military or emergency parliament. (statement l is incorrect)
powers like the president Parliament acts as a investigative body against all the
Important Tip decisions taken by the executive Legislature Ministry and
even the duties of governor.
The Governor is the constitutional head of the State
Government. Important Tip
He plays a two-fold function as the constitutional head Article 153 to Article 162 of the Indian Constitution
of the State Government and as a link between the consists of the relevant provisions related to the office
Centre and the state government. of the Governor.
The Governor of a State is appointed by the President The Governor is the constitutional head of the State
by warrant under his hand and seal. Government
Articles 153 says that three should be a Governor for appointed by the President by warrant under his hand
each state. and seal.

l l l e - - - - - - - - - - - - - - - - - - - - - - - - - - - - - - - - - - - - - - P Y Q Workbook

KING R QUEEN P [ऋषभ राजपूत ]


II INDIAN POLIT): AND~OVERNANCE II
Articles 153 says that three should be a Governor for The Electoral College for electing the President of
each state. India consists of the following:
under the 7th Amendment Act, 1956, the same person All elected members of the Rajya Sabha
can be appointed as Governor of one or more States. All elected members of the Lok Sabha
Article 157: In order to be appointed as Governor, a All elected members of the Legislative Assemblies
person: of States
Must be a citizen of India; All elected members of the Legislative Assemblies
Must have competed the age of 35 years of Union Territories (except Jammu and Kashmir)
In addition, three are two conventions that have come
131. Solution: (b)
to develop with regard to appointment of the Governor.
Exp) Option b is the correct answer.
Must not belong to the state where he is appointed
and Consult the Chief Minister of the state where If any question arises regarding the disqualification of a
to be appointed member of Parliament, it shall be referred to the President
for decision. However, the President's decision is not
129. Solution: (a) discretionary in this matter. The President is required to
Exp) Option a is the correct answer. obtain the opinion of the Election Commission and act
The following Prime Ministers were defeated by a vote of according to that opinion. Therefore, this power does not
no confidence motion: come under the President's discretion.
a) Morarji Desai: 1978 132. Solution: (a)
b) Vishwanath Pratap Singh : 1990 Exp) Option a is the correct answer.
c) HD Deve Gowda :1997 Statement 1 is correct- The governor may exercise
d) Atal Bihari Vajpayee :1999 discretionary powers, ifhe/ she is not convinced by the advice
No confidence motion: (Rule 198 Lok Sabha) of the Council of Ministers. For example, the Governor may
• Introduced only in Lok Sabha refuse to give assent to a bill passed by the state legislature if
he/she believes that the bill is unconstitutional or otherwise
Supported by at least 50 members
harmful to the interests of the state.
• Debate
Statement 2 is correct- The Constitution of India specifies
Not able to prove Majority Government has to resign certain matters in which the Governor has discretionary
(Simple Majority) powers which includes appointing the Chief Minister of the
130. Solution: (c) state, approving the appointment of ministers, dissolving
the state legislature, and giving assent to bills passed by
Exp) Option c is the correct answer.
the state legislature
Statement 1 is correct- The Rajya Sabha, also known as
Statement 3 is correct- In cases where the Governor is
the Council of States, is the upper house of the Parliament
appointed as the administrator of an adjoining Union
of India. Elected members of the Rajya Sabha are part
of the Electoral College and have the right to vote in the Territory, they have the power to exercise discretionary
presidential election. powers in the administration of that territory.

Statement 2 is correct- All elected members of the 133. Solution: (b)


Legislative Assemblies of States are included in the Exp) Option b is the correct answer.
Electoral College. The Legislative Assembly of each state
in India consists of elected representatives who have the The Attorney-General of India is the highest law officer of
authority to vote in the presidential election. the government and provides legal advice to the government.
The Attorney-General of India enjoys the right of audience
Statement 3 is incorrect- The Legislative Council, also
in not only the Supreme Court but also in any court
known as the Vidhan Parishad, is the upper house of
within the territory of India, including High Courts and
the state legislature in some states of India. However, the
subordinate courts.
members of the Legislative Councils are not part of the
Electoral College for electing the President of India. Important Tip
Important Tip Some other facts about Attorney-General of India:
Election of President of India: The position of Attorney General is created by Article
The election is held in accordance with the system 76 of the Indian Constitution.
of proportional representation by means of a single The Attorney General is appointed by the President
transferable vote. Voting is done through Secret of India and holds office during the pleasure of the
ballot. President.

PYQ W o r k b o o k - - - - - - - - - - - - - - - - - - - - - - - - - - - - - - - - - - - - - 1 1 1

KING R QUEEN P [ऋषभ राजपूत ]


II II
December 1946 and served in that capacity until the
To be eligible for the position, the person must be
qualified to be appointed as a judge of the Supreme Constituent Assembly was dissolved on 24 January 1950.
Court. Prasad was a prominent leader of the Indian independence
movement and a close associate of Mahatma Gandhi. He was
The remuneration of the Attorney General is
also the first President of India from 1950 to l 962
determined by the President.
The Attorney General can participate in the 137. Solution: (d)
proceedings of Parliament but does not have the right Exp) Option dis the correct answer
to vote.
The Attorney-General of India, as the highest law officer
The Attorney General can be removed from office by
of the government under Article 76, is not exclusively
the President at any time.
engaged as a whole-time counsel for the government. The
134. Solution: (b) Attorney-General is free to engage in private legal practice
Exp) Option b is the correct answer. or pursue other professional responsibilities outside the
realm of official duties.
According to Article 258(1) of the Indian Constitution,
the President of India has the power to entrust, with the Important Tips
consent of the Government of a State, certain functions to To avoid conflict of duty, there are a few limitations that
that Government or its officers in relation to any matter to are posted on the Attorney General which he should
which the executive power of the Union extends. keep in mind while performing his duties:
135. Solution: (c) He should not advise or hold a brief against the
Exp) Option c is the correct answer. Government of India

Statement 1 is correct- According to Article 123, the He should not advise or hold a brief in cases in
President has the power to promulgate an ordinance when which he is called upon to advise or appear for the
either one or both houses of Parliament are not in session. Government of India
These ordinances are temporary in nature and can be issued He should not defend accused persons in criminal
to address urgent matters. prosecutiom without the permission of the
Statement 2 is correct- The approval of Parliament is Government of India
necessary for an ordinance to become a law. The ordinance He should not accept appointment as a director in
must be laid before both houses of Parliament and must any company or corporation without the permission
be approved within six weeks from the reassembly of of the Government of India
Parliament. If not approved, the ordinance ceases to have
138. Solution: (b)
effect.
Exp) Option b is the correct answer
Important Tip
The Thakkar Commission was appointed in 1984 to
Ordinances can only be introduced on subjects within investigate the assassination of Indira Gandhi, to find
the legislative competence of the Indian Parliament.
The President's ordinances have the same effect on
policies as acts passed by Parliament.
lapses by the security and medical staff, and to determine
possible involvement of foreign agencies. I
139. Solution: (d)
Article 213 deals with the power of the Governor to
legislate through ordinances. Exp) Option dis the correct answer
The Governor's ordinances have the same effect on The power of pardon is a discretionary power vested in
policies as state acts will have. the President, and it is independent of the judiciary. The
If the Governor's ordinance legislates on matters where President has the authority to exercise this power and grant
the state government has no power, the ordinance pardon even in cases where there is a judicial mandate or
becomes null and void. decision. The power of pardon is meant to provide relief,
The power to promulgate ordinances by the President mercy, or mitigation in certain cases, and it is not restricted
and the Governor is not discretionary; it requires the by the judicial mandate.
advice of the Council of Ministers, headed by the
Important Tips
Chief Minister in the case of the state government
and Prime Minister in the case of Union government. Under Article 72 of the Constitution, the President
shall have the power to grant pardons, reprieves,
136. Solution: (b) respites or remissions of punishment or to suspend,
Exp) Option b is the correct answer. remit or commute the sentence of any person
convicted of any offence where the sentence is a
Rajendra Prasad was the first President of the Constituent
sentence of death.
Assembly of India. He was elected as the President on 11

111,--------------------------------------PYQWorkbook

KING R QUEEN P [ऋषभ राजपूत ]


II INDIAN POLITY AND GOVERNANCE II
143. Solution: (a)
Limitation:
Exp) Option a is the correct answer.
• The President cannot exercise his power of pardon
independent of the government. Statement 1 is correct: The Preamble of the Constitution
In several cases, the Supreme Court has ruled that the of India is a brief introductory statement that sets out the
President has to act on the advice of the Council of guiding principles and purpose of the Constitution. It is
Ministers while deciding mercy pleas. based on the Objective Resolution, which was moved by
Jawaharlal Nehru in the Constituent Assembly on 13
Although the President is bound by the Cabinet's
December 1946 and adopted on 22 January 1947.
advice, Article74 (1) empowers him to return it for
reconsideration once. If the Council of Ministers Statement 2 is correct: The idea of the preamble was
decides against any change, the President has no borrowed from the Constitution of the USA.
option but to accept it. Statement 3 is correct: The Preamble declares India to be a
sovereign, socialist, secular, democratic republic and secures
140. Solution: (a) to all its citizens justice, liberty, equality and fraternity.
Exp) Option a is the correct answer. The words socialist and secular were added by the 42nd
Under Article 355 of the Constitution of India, it is the Amendment in 1976.
duty of the Union to protect every State against external Statement 4 is incorrect: While the Preamble of the
aggression and internal disturbance. This means that the Constitution of India sets out the ideals and objectives
Union government is responsible for ensuring the security of the Constitution, it does not explicitly mention the
and integrity of the States by defending them against any Fundamental Rights. The Fundamental Rights are enshrined
external threats or acts of aggression. in Part III of the Constitution.

141. Solution: (d) 144. Solution: (b)


Exp) Option d is the correct answer. Exp) Option b is the correct answer.

The 2nd Administrative Reforms Commission (ARC) in Statement 1 is correct: The President of India is elected
India has produced Reports on various subjects aimed at indirectly, which means that the citizens of India do not
improving governance and administrative processes. The directly vote for the President. Instead, the President is
Reports produced by the ARC are: elected by an electoral college.
Statement 2 is correct: The electoral college for the
Ethics in Governance: The ARC has addressed the issue
Presidential elect.ion comprises all elected Members of
of ethics in governance, emphasizing the need for ethical
Parliament (MPs) from both the Lok Sabha and the Rajya
conduct and integrity in public administration.
Sabha, as well as Members of Legislative Assemblies
Local Governance: The ARC has focused on the issue of (MLAs) from all the states and Union territories. These
local governance, recommending reforms to strengthen members collectively vote to elect the President of India.
decentralization and empower local bodies for effective
Statement 3 is incorrect: President of India is elected
service delivery and citizen participation.
indirectly, not through a direct vote by the citizens.
Combating Terrorism: The ARC has examined the
Statement 4 is correct: Members of the Legislative Council,
challenges posed by terrorism and made recommendations
which is the upper house of the state legislature in certain
to enhance the capacity of security agencies, improve
states, do not participate in the Presidential election. Only
intelligence sharing, and strengthen counter-terrorism Members of Parliament (MPs) and Members of Legislative
measures. Assemblies (MLAs) have the right to vote in the Presidential
The 2nd ARC did not produce a report on eradication of election.
corruption specifically, though it touched upon the issue
in some of its reports such as ethics in governance and 145. Solution: (a)
refurbishing personnel administration. Exp) Option a is the correct answer.
Statement I is correct: According to Article 153 of the
142. Solution: (a)
Constitution of India, there can be a common Governor
Exp) Option a is the correct answer. for two or more states. This provision allows for the
According to Article 53 of the Indian Constitution, the appointment of a Governor who can act as the Governor of
President of India is the Supreme Commander of the multiple states.
Indian Armed Forces. The President holds the highest rank Statement 2 is correct: The Governor has the power to
and authority over the military forces of India, including the grant pardons, reprieves, and remission of punishment
Indian Army, Indian Navy, and Indian Air Force. to persons convicted under the State Law. This power is
The President exercises the powers and functions as the mentioned in Article 161 of the Constitution of India.
Supreme Commander in consultation with the Council of Statement 3 is incorrect: The Governor is not appointed
Ministers, headed by the Prime Minister. by the President on the recommendation of the Chief

PYQ W o r k b o o k - - - - - - - - - - - - - - - - - - - - - - - - - - - - - - - - - - - - - - < 1 1 1
l!Forum~

KING R QUEEN P [ऋषभ राजपूत ]


II INPIANPOUTY AND GOVERNANCE II
Minister. The Governor is appointed by the President of procedures of the government and defines its relationship
India, and the appointment is made by the President based with the people. The constitution also lays down the
on the advice of the Union Council of Ministers. The Chief principles and values that guide the actions and decisions of
Minister of the respective state does not have a role in the the government and its officials.
appointment of the Governor. Statement 3 is incorrect: A constitutional government does
146. Solution: (a) not necessarily have a monarch as the formal head of the
state. A constitutional government can have different forms
Exp) Option a is the correct answer.
of executive leadership, such as a president, a prime minister,
Statement l is correct: Constitutional government is a form or a chancellor.
of limited government. The constitution sets the boundaries
Statement 4 is incorrect: A constitutional government does
and limits of the powers of the government and prevents it
not always have a universal adult franchise. A constitutional
from becoming arbitrary or tyrannical. The constitution also
government can have different forms of electoral systems,
ensures that the government is accountable to the people and
such as proportional representation, first-past-the-post, or
respects their rights and freedoms.
mixed-member proportional. A universal adult franchise is
Statement 2 is correct: A constitutional government is one one of the possible ways of ensuring political participation
where the constitution is the basis of public authority. and representation in a constitutional democracy, but it is
The constitution establishes the structure, functions, and not a necessary condition for a constitutional government .

••• ,.

1 1 1 ~ - - - - - - - - - - - - - - - - - - - - - - - - - - - - - - - - - - - - P Y Q Workbook
PDForun11Y~

KING R QUEEN P [ऋषभ राजपूत ]


- ' -
lfflffll~ffl 1~131~~ ~fflffl ®~~lf,Niffl~~~m
UNION AND STATE LEGISLATURE
*This unit consists of questions from Union Legislature and State Legislatures.

di (JPSCCSE Ptevi<>us Years'Qu.estiorts of Lok Sabha shall be from either the


principal opposition party or the ruling
1. With reference to Finance Bill and Money party.
Bill in the Indian Parliament, consider the 3. The Deputy Speaker has the same power
following statements: as of the Speaker when presiding over the
1. When the Lok Sabha transmits Finance sitting of the House and no appeal lies
Bill to the Rajya Sabha, it can amend or against his rulings.
reject the Bill. 4. The well-established parliamentary
2. When the Lok Sabha transmits Money practice regarding the appointment of
Bill to the Rajya Sabha, it cannot amend Deputy Speaker is that the motion is
or reject the Bill, it can only make moved hy the Speaker and duly seconded
recommendations by the Prime Minister.
3. In the case of disagreement between the Which of the statements given above are
Lok Sabha and the Rajya Sabha, there is correct?
no joint sitting for money bill, but joint (a) 1 and 3 only
sitting becomes necessary for Finance (b) 1, 2 and 3
bill. (c) 3 and 4 only
How many of the above, statements are (d) 2 and 4 only
correct? [UPSC CSE Pre Pre 2023]
(a) Only one
4. With reference to anti-defection law in
India, consider the following statements:
(b) Only two
( c) All three 1. The law specifies that a nominated
(d) None legislator cannot join any political party
within six months of being appointed to
2. Which of the following is/are the exclusive the House.
power(s) of Lok Sabha? 2. The law does not provide any time-frame
[UPSC CSE Pre. 2022] within which the presiding officer has to
1. To ratify the declaration of Emergency decide a defection case.
2. To pass a motion of no-confidence against Which of the statements given above is/are
the Council of Ministers correct? [UPSC CSE Pre Pre 2022]
3. To impeach the President of India (a) 1 only
Select the correct answer using the code given (b) 2 only
below: (c) Both 1 and 2
(a) 1 and 2 (d) Neither 1 nor 2
(b) 2 only 5. Consider the following statements:
(c) 1 and 3
1. A bill amending the Constitution requires
(d) 3 only
a prior recommendation of the President
3. With reference to Deputy Speaker of Lok of India.
Sabha, consider the following statements: 2. When a Constitution Amendment Bill is
[UPSC CSE Pre. 2022] presented to the President of India, it is
1. As per the Rules of Procedure and obligatory for the President of India to
Conduct of Business in Lok Sabha, the give his/her assent.
election of Deputy Speaker shall be held 3. A Constitution Amendment Bill must
on such date as the Speaker may fix. be passed by both the Lok Sabha and the
2. There is a mandatory provision that the Rajya Sabha by a special majority and
election of a candidate, as Deputy Speaker there is no provision for joint sitting.

PYQ W o r k b o o k - - - - - - - - - - - - - - - - - - - - - - - - - - - - - - - - - - - 1 1 1 1

KING R QUEEN P [ऋषभ राजपूत ]


II II
196. Solution (c) Additional disqualifications specified in the
Exp) Option c is the correct answer. Representation of People Act (1951) include
Insurance is a subject that comes under the Union List convictions for election offenses, corrupt practices,
(Entry 43 and 47) of the Indian Constitution. The central imprisonment of two or more years, failure to
submit election expense accounts, and having an
government has the authority to make laws related to
interest in government contracts or corporations with
insurance.
government shareholding.
Important Tips Disqualifications also apply to individuals dismissed
State List: from government service for corruption, convictions
It contains subjects on which the state governments for promoting enmity between groups or bribery,
have the exclusive power to make laws and regulations. and punishments related to social crimes such as
untouchability, dowry, and sati. The President's
It includes matters such as public health, police, public
decision, based on the Election Commission's opinion,
order, local government, agriculture, state taxes, land
is final in determining disqualifications.
and land reforms, education, and public libraries.
The State List consists of a total of 59 subjects listed 198. Solution (a)
under Schedule VII of the Indian Constitution. Exp) Option a is the correct answer.
Union List: Statement l is correct - An overseas elector must be a citizen
The Union List contains subjects on which the central of India to be eligible for voting in the Lok Sabha elections.
government has exclusive authority to make laws and Statement 2 is correct - The name of the overseas elector
regulations. must be included in the electoral roll of the constituency
It includes matters such as defense, foreign affairs, they would be voting from to be eligible to vote.
atomic energy, banking, currency, railways, airways, Statement 3 is correct - Overseas Indian voters are required
communication, and inter-state trade and commerce. to be physically present in India for voting, as e-voting is not
The Union List consists of a total of 98 subjects listed allowed. They have the options of postal ballots and proxy
under Schedule VII of the Indian Constitution. voting, with certain restrictions and eligibility criteria.
Concurrent List:
199. Solution (c)
The Concurrent List contains subjects on which both
Exp) Option c is the correct answer
the central and state governments have the power to
make laws and regulations. The Secretary General of the Lok Sabha does not work under
the Speaker with delegated authority. Instead, the Secretary
It includes matters such as criminal law, marriage
General works independently and performs various
and divorce, bankruptcy and insolvency, adoption,
administrative functions related to the functioning of
educntion, forests, trade unions, bankruptcy and
the Lok Sabha. The Secretary General is the principal
insolvency, and bankruptcy and insolvency.
administrative officer of the Lok Sabha Secretariat and is
The Concurrent List consists of a total of 52 subjects responsible for the overall management and coordination of
listed under Schedule VII of the Indian Constitution.
its activities.
197. Solution (b) Important Tips
Exp) Option b is the correct answer. In the discharge of his constitutional and statutory
According to Article 102 of the Indian Constitution, certain responsibilities, the Speaker of the Lok Sabha is assisted
conditions can lead to the disqualification of a Member of by the secretary general of Lok Sabha, (whose pay
Parliament. Option (b) Holding the office of the Chairperson scale, position and status etc. is equivalent to that
of the highest-ranking official in the Government of
of the National Commission for Women does not come
India i.e. Cabinet Secretary).
under the office of profit.
The secretary general remains in office must retire
Important Tips at the age of 60. The secretary general is answerable
Factors leading to disqualification from being elected as a only to the Speaker; actions cannot be discussed or
Member of Parliament (MP) in India: cdtidsed in or outside the Lok Sabha.
Holding any office of profit under the Union or state On behalf of the President, the secretary-general
government, except for exempted offices. summons each Member of Lok Sabha to attend
Being declared of unsound mind by a court or being an session of the Parliament and authenticates bills in
undischarged insolvent. the absence of the Speaker.
Acquiring citizenship of a foreign state voluntarily or 200. Solution (b)
having allegiance to a foreign state. Exp) Option bis the correct answer
----------~

KING R QUEEN P [ऋषभ राजपूत ]


II
Statement I is correct: The privileges of Members of In a state, the state legislative council is considered as the
Parliament (MPs) are separate from the fundamental upper house of the state which have a bicameral legislature
rights guaranteed under Article 19(l)(a). The privileges of and the state Legislative assembly is called as lower
MPs are defined by the Constitution and laws passed by house house of the state. One third of the Members of
the Parliament, and they are not subject to the limitations legislative council are elected by electorate consisting
imposed by Article 19(l)(a). oflocal bodies and authorities (option c is incorrect)
Statement 2 is correct: While the privileges of MPs are At present there are 6 out of 28 states have a legislative
protected, they are subject to certain limitations. Articles Council.
20-22 of the Indian Constitution provide specific rights and
The members of the Legislative council can't be more
protections regarding protection against self-incrimination,
than one-third of the membership of the state legislative
protection against double jeopardy, and protection of
assembly and its composition can't be less than 40
personal liberty. Article 32 allows individuals to approach
members.
the Supreme Court for the enforcement of their fundamental
rights. Therefore, the privileges of MPs must be read subject Important Tips
to these constitutional provisions. India has a bicameral system of legislatures both at the
Statement 3 is incorrect: The privileges of MPs grant them centre and in some states.
immunity from civil proceedings only and not from criminal At the state level, the equivalent of the Lok Sabha is
prosecution. MPs are not immune from criminal charges the Vidhan Sabha or Legislative Assembly and that of
and can be prosecuted for offenses committed like any other the Rajya Sabha is the Vidhan Parishad or Legislative
citizen. Council.
Statement 4 is incorrect: While MPs enjoy certain Article 169 provides for creation and abolition of
protections for their speech and conduct within Parliament, legislative councils in states.
these privileges do not extend to their private or personal
capacity outside of parliamentary proceedings. Outside of 203. Solution (d)
Parliament, MPs are subject to the same laws and regulations Exp) Op ton d is the correct answer
as any other citizen, and their freedom of speech is governed Rajya Sabha being a federal chamber - representing States/
by the general laws of the land. Union territories, enjoys certain special powers:
Important Tips a) To empower Parliament to make laws in respect of any
Individual Privileges of the Members of the Parliament matter enumerated in the State List in the national
interest by adopting a resolution to this effect (article
No arrest of the member of the parliament can take
249), (option dis correct)
place during its session. Also, members can't be
arrested 40 days before and after the session's beginning b) Creation of All India Services (article 312)
and end of the session. c) Approving Proclamations (issued under article 352 or
The members of parliament are entitled to the article 356 or article 360) if the Lok Sabha stand dissolved
freedom of speech in the houses. They are not liable or the dissolution of the Lok Sabha takes place within the
to any court proceedings for the speech given in the period allowed for the approval of the Proclamation by
parliament or its committees. However, it is regulated Parliament.
using the rules guiding such provisions of the house. d) To Initiate the proceeding for removal of Vice President
They are exempted from jury service. They can refuse is also the exclusive jurisdiction of Rajya Sabha i.e. The
to give evidence and appear as a witness in a case resolution for removal of Vice President can only be
pending in a court when Parliament is in session. initiated in Rajya Sabha and not in Lok Sabha.
201. Solution (d) Important Tips
Exp) Option d is the correct answer The Council of States (Rajya Sabha) was First Constituted
The process of appointing Members of Parliamentary on 3rd April, 1952.
Committees involves a combination of appointment, The Council of States as Rajya Sabha in Hindi on 23rd
election, and nomination. In the case of some committees, August, 1954.
the Members are appointed or elected through a motion Rajya Sabha is a permanent and a continuous chamber
made and adopted by the Speaker or Chairman. In other where representatives are elected for the house.
cases, Members may be nominated directly by the Speaker The First Chairman ofRajya Sabha was Dr. S. Radhakrishnan.
or Chairman without the need for a motion. Nominations Dr. S. Radhakrishnan was the first Chairman of
are made based on various considerations such as party Rajya Sabha for two consecutive terms (13.5.1952 to
representation, expertise, and experience. 12.5.1962).
202. Solution (c) The First Deputy Chairman of Rajya Sabha was Shri
S.V. Krishnamoorthy Rao.
Exp) Option c is the correct answer.

PYQ W o r k b o o k - - - - - - - - - - - - - - - - - - - - - - - - - - - - - - - - - - - - - 1 1 1

KING R QUEEN P [ऋषभ राजपूत ]


II II
204. Solution (a) budget year in three separate parts viz. Consolidated
Exp) Option a is the couect answer Fund of India, Contingency Fund of India and Public
Account of India.
To discuss state matters in general condition does not lie
under procedure and conduct of business of Parliament 208. Solution (d)
under Article 246 of the Constitution. (option a is not
Exp): Option dis the correct answer.
correct)
Parliament has right to make laws on Union list and Third It contains the forms of oath and affirmation
Concurrent list of Schedule seven. Schedule for:
Parliament can discuss the issue of Police action against Union Ministers of India
the Schedule Castes and Tribes under the Scheduled Parliament Election Candidates
Castes and Tribes (Prevention of Atrocities) Act, 1989
Members of Parliament (MPs)
Issue of Industrial labours come under Concurrent list,
so it can be discussed in Parliament. Supreme Court Judges
Parliament can also discuss issues in dealing with violent Comptroller and Auditor General
disturbances in an undertaking under the control of the State Ministers
Union Government State Legislature Elections' Candidates
Important Tips State Legislature Members
Rules of Procedure and Conduct of Business in the High Court Judges
Council of States have been formulated under Article 118
Fourth It contains the provisions in relation to
of the Constitution wherein each House of Parliament is
Schedule the allocation of seats for States and Union
required to make rules for regulating its Procedure and
Territories in the Rajya sabha (option d is •
conduct of business.
correct)
205. Solution (d) Fifth It contains provisions in relation to the
Schedule administration and control of scheduled areas
.,.
Exp) Option dis the correct answer
and scheduled tribes
The legislature of states involve Governor, Legislative
Assembly and the Legislative Council. Sixth It contains provisions in relation to the
Under article 164 of the Indian constitution chief • Schedule administration of tribal areas in the states of
minister shall be appointed by the governor. Assam, Meghalaya, Tripura and Mizoram

Important Tips 209. Solution (b)


Part VI of the Constitution containing Articles 153 to Exp) Option b is the correct answer.
167 deals with the government in the States. According to article l 02 of the constitution of India:
The state executive consists of the Governor, the Chief 1. A person shall be disqualified for being chosen as, and for
Minister, the Council of Minister and the Advocate being, a member of either House of Parliament:
General of the state.
(a) if he holds any office of profit under the Government of
206. Solution (d) India or the Government of any State, other than an office
Exp) Option dis the correct answer. declared by Parliament by law not to disqualify its holder

According to article 108 a joint sitting is called by the (b) if he is of unsound mind and stands so declared by a
President and presided over by the speaker. competent court;

A bill which requires special majority in both the houses (c) ifhe is an undischarged insolvent;
cannot be introduced in joint sitting of the Parliament (d) if he is not a citizen of India, or has voluntarily acquired
Even for money bill Rajya sabha can only amended some the citizenship of a foreign State, or is under any
changes and it is Lok sabha only which passes the money acknowledgement of allegiance or adherence to a foreign
bill. Hence there is no point in introducing the money State;
bill in the joint sitting of the Parliament (e) if he is so disqualified by or under any law made by
Parliament Explanation
207. Solution (c)
2. A person shall be disqualified for being a member of
Exp) Option c is the correct answer.
either House of Parliament if he is so disqualified under
Annual Financial Statement is a document presented to the Tenth Schedule.
the Parliament every Financial Year as a part of Budget
process under Article 112. Important Tips

This document comprises the receipts and expenditures Disqualification of MPs & MLAs according to
of the government of current year, previous year and representation of people act 1951:

l l l f - - - - - - - - - - - - - - - - - - - - - - - - - - - - - - ~ - - - - - - - P Y Q Workbook

KING R QUEEN P [ऋषभ राजपूत ]


II
Statement 3 is incorrect: State Election Commission is 26. Solution: (b)
mandated to Conducts elections only for Panchayats and Exp) Option bis the correct answer.
Municipalities of the State. The committee which recommended a three-tier Panchayati
Statement 4 is incorrect: This function (preparation of the Raj System in India was the Balwant Rai Mehta Committee.
electoral rolls and conducts elections) is not assigned to
Important Tips
the State Election Commission, but to the Chief Electoral
Officer of each state, who works under the supervision and The Balwant Rai Mehta Committee was appointed
control of the Election Commission of India. The Chief in 1957 by the Government of India to examine and
suggest measures for better working of the Community
Electoral Officer is responsible for preparing and revising
Development Programme and the National Extension
the electoral rolls and conducting elections for Parliament
Service.
and state legislatures, as per Article 324 of the Constitution.
The committee submitted its report in 1958, and
24. Solution: (b) recommended the establishment of a democratic
Exp) Option b is the correct answer. decentralised local government, which came to be known
as the Panchayati Raj.
The Gram Panchayat bears the responsibility of planning,
The committee suggested a three-tier system of Panchayati
execution, and implementation of MNREGA (Mahatma
Raj, comprising the following:
Gandhi National Rural Employment Guarantee Act) schemes
at the local level. Gram Panchayat at the village level, directly elected by the
people.
Important Tips Panchayat Samiti at the block level, indirectly elected by
Features of MGNREGA: the members of the Gram Panchayats within the block.
It is a scheme that provides legal guarantee of wage Zila Parishad at the district level, indirectly elected by the
employment to the adult members of rural households. members of the Panchayat Samitis within the district.
• Every rural household has the right to register 27. Solution: (d)
under MGNREGA, and job cards are issued to every
Exp) Option d is the correct answer.
household registered under MGNREGA within 15 days
from the date of receipt of the application for the job The state government controls over urban local bodies in
card registration. legislative, financial, and personnel matters. This means
that:
A minimum of 10 job seekers shall apply to sanction a
new work under MGNREGA The state government can enact laws and regulations
regarding the functioning, powers, and finances of the
The work shall be provided within 5 km jurisdiction urban local bodies, such as municipalities and municipal
from the village. If the work provided is beyond 5 Kms, corporations. The urban local bodies can only make rules
the job seekers shall be given 10% of the minimum or by-laws within the framework of the state laws.
wages as the additional amount
The state government can determine the sources and
• Social Audit of MGNREGA works is mandatory, which methods of revenue generation for the urban local
lends to accountability and transparency bodies, such as taxes, fees, grants, loans, etc. The state
government can also regulate the expenditure and audit
25. Solution: (a)
of the urban local bodies, and impose financial discipline
Exp) Option a is the correct answer, and control on them.
Charles Metcalfe was an acting Governor-General of India The state government can decide the recruitment,
from March 1835 to March 1836 and he wrote about the self- training, promotion, transfer, discipline, and service
sufficiency and vitality of the Indian village communities. conditions of the employees of the urban local bodies.
He said, "The village communities are little republics, having The state government can also prescribe the qualifications
nearly everything that they want within themselves, and and pay scales of the employees of the urban local bodies.
almost independent of any foreign relations." The state government does not control over urban
Important Tips local bodies in respect of citizens' grievances. This
means that:
Metcalfe (governor general-1835-36) repealed the
The state government cannot interfere with the
obnoxious 1823 ordinance (using a press without license
complaints or grievances of the citizens against the
was a penal offence) and earned the epithet, "liberator of
urban local bodies or their functionaries. The urban
the Indian press".
local bodies are responsible for addressing and redressing
The new Press Act (1835) required a printer/publisher to the grievances of the citizens within their jurisdiction.
give a precise account of premises of a publication and
The state government can only appoint ombudsmen
cease functioning, if required by a similar declaration. or lokayuktas to investigate and redress the grievances

PYQ W o r k b o o k - - - - - - - - - - - - - - - - - - - - - - - - - - - - - - - - - - - - - 1 1 1

KING R QUEEN P [ऋषभ राजपूत ]


II
of the citizens against the urban local bodies or their government bodies established in large urban areas to
functionaries. The ombudsmen or lokayuktas are administer and manage the complex affairs of the city.
independent and impartial authorities who act as Reason (R) is correct: Urban administrations of big cities
watchdogs over the urban local bodies. are very complex in nature. This statement is also true.
28. Solution: (a) Big cities have dense populations, multiple infrastructure
needs, various social and economic complexities, and
Exp) Option a is the correct answer.
require specialized governance structures to address these
Pair A is correctly matched: The Committee on challenges effectively.
Rationalization of Panchayat Statistics was appointed by the
Government of India in 1960 under the chairmanship of V.R. 31. Solution: (d)
Rao Exp) Option d is the correct answer.
Pair B is incorrectly matched: A.S. Trivedi was not According to the Panchayati Raj system in India, a Gram
associated with Panchayat Raj Training. S.P. Jain was Panchayat is the elected body that governs a village or a
involved in developing an action plan for training of group of villages, while a Gram Sabha is the assembly of all
Panchayati Raj functionaries adult members of a village or a group of villages
Pair C is correctly matched: The Working Group on Statement 1 is correct: In a Gram Panchayat there can be
Panchayats and Cooperatives was constituted by the more than one village. This is correct, as a Gram Panchayat
Government of India, Ministry of Community Development may consist of one or more villages, depending on the
and Cooperation in 1961 under the chairmanship of S.D. population and area
Mishra Statement 2 is correct: Meeting of Gram Sabha held in
Pair D is correctly matched: The Committee to review each village. This is correct, as the meeting of Gram Sabha
the existing Administrative Arrangements for Rural is held at least twice a year in each village within the area of
Development and Poverty Alleviation Programmes under the the Gram Panchayat
chairmanship of G.V.K. Rao was appointed by the Planning Statement 3 is incorrect: Meeting of Gram Sabha held in
Commission in 1985 the headquarter of Gram Panchayat. This is incorrect, as the
meeting of Gram Sabha is not held in the headquarter of
29. Solution: (d)
Gram Panchayat, but in each village under its jurisdiction.
Exp) Option dis the correct answer.
Statement 4 is incorrect: Meeting of Gram Panchayat held
Option I is incorrect: 74th Amendment Act does not in each village. This is incorrect, as the meeting of Gram
mention municipal committees as a type of urban local Panchayat is not held in each village, but in the headquarter
government. It only mentions three types of municipalities: of the Gram Panchayat or any other place fixed by it.
nagar panchayats, municipal councils and municipal
Statement 5 is correct: Meeting of Gram Panchayat held
corporations.
in the headquarter of Gram Panchayat. This is correct, as
Option II is correct: 74th Amendment Act provides for the the meeting of Gram Panchayat is usually held in the
constitution of ward committees at ward level or other headquarter of the Gram Panchayat or any other place fixed
level within the territorial area of a municipality as may be by it
provided in the state law.
Option III is correct: 74th Amendment Act provides for 32. Solution: (c)
the constitution of a metropolitan planning committee Exp) Option c is the correct answer.
for every metropolitan area, which is an area having Statement l is incorrect: Polling shall be postponed. This
a population of ten lakhs or more, to prepare a draft statement is false because polling shall not be postponed in
development plan for the area. all cases of death of a candidate before polling. It depends
Option IV is correct: 74th Amendment Act provides for on whether there are other contesting candidates in that seat
the constitution of a district planning committee for every or not.
district, to consolidate the plans prepared by the panchayats Statement 2 is incorrect: Polling shall not be postponed.
and the municipalities in the district and to prepare a draft This statement is false because polling shall be postponed in
development plan for the district as a whole some cases of death of a candidate before polling. It depends
on whether there are other contesting candidates in that seat
30. Solution: (a)
or not.
Exp) Option a is the correct answer.
Statement 3 is correct: If only one candidate remains, then
Both Assertion (A) and Reason (R) are true, and Reason polling will be postponed. This statement is true because if
(R) is correct explanation of Assertion (A). a candidate dies before the polling date and there is no other
Assertion (A)is correct: Municipal corporations are formed contesting candidate in that seat, then the election for that
in big cities having a definite area and population. This seat will be countermanded and fresh election will be held
statement is true. Municipal corporations are local self- later.

fZllc---------------------------------------PYQWorkbook
IJForum!m

KING R QUEEN P [ऋषभ राजपूत ]


II
Statement 4 is correct: If more than one candidate remains, accommodate changes in the demographic, administrative,
then polling will not be postponed. This statement is true or developmental requirements of the region. These changes
because if a candidate dies before the polling date and there could include the creation of new Blocks or Zila Panchayats,
are other contesting candidates in that seat, then the election the merger of existing ones, or alterations in their territorial
for that seat will not be countermanded and polling will be jurisdiction.
held as per schedule.
36. Solution: (d)
33. Solution: (b) Exp) Option d is the correct answer.
Exp) Option b is the correct answer. In such a case, the state government may have the primary
Statement 1 is incorrect and Statement 2 is correct: responsibility to resolve the dispute, as it has administrative
The term begins from the date of the first meeting of the control over the panchayats and their areas. However, the
Municipal Corporation. state government may also need the approval of the central
Statement 3 is correct: The tenure of the Municipal Council government, as it has legislative and executive control over
is 5 years the cantonments and their boards.
Statement 4 is correct: A municipality constituted upon Important Tips
the dissolution of a municipality before the expiration of its A panchayat is a local self-government institution under
duration shall continue only for the remainder of the period the 73rd Constitutional Amendment Act, 1992. It has
for which the dissolved municipality would have continued powers and functions as per the state laws and the Eleventh
had it not been so dissolved. If the municipal council is Schedule of the Constitution.
dissolved within the first two years of its tenure, a new A cantonment board is a statutory body under the
municipal council will be organized for the remaining three Cantonments Act, 2006. It has powers and functions as per
years of its tenure. the central laws and the rules and regulations made by the
Statement 5 is incorrect: A municipality reconstituted after central government.
premature dissolution does not enjoy the full period of
37. Solution: (e)
five years but remains in office only for the remainder of
the period. Exp) Option e is the correct answer.
The Zila Panchayats of Scheduled Areas have the powers
34. Solution: (c) to:
Exp) Option c is the correct answer. (a) Plan minor water bodies: They can plan and undertake
Statement 1 is correct: The wards committee is formed projects related to minor water bodies within their
with some wards of nagar Panchayat: This statement is jurisdiction.
correct because a nagar panchayat is an urban local body that (b) Control over institutions in all social sectors: Zila
administers a town with a population of 100,000 or less, and Panchayats have the authority to oversee and manage
it is divided into wards according to population. iii.stitutions in various social sectors, including education,
Statement 2 is correct: Councillor elected from the wards health, and other welfare activities.
become its member: This statement is correct because the (c) Control on tribal sub plans: They have the responsibility
members of the wards committee are elected by direct to implement and monitor tribal sub-plans, which are
election from electoral wards in the nagar panchayat for a designed to ensure the socio-economic development of
term of five years. tribal communities.
Statement 3 is correct: Two persons are nominated in the (d) Exercise other functions as conferred by the State
committee residing in the ward: This statement is correct Government: In addition to the above-mentioned
because in addition to the elected members, there are three powers, Zila Panchayats may be entrusted with other
nominated members in the wards committee, who are functions by the State Government as per the specific
residents of the ward. requirements and policies of the state.
Statement 4 is incorrect: The Chairperson of the Wards
38. Solution: (d)
Committee is elected by the elected members thereof from
among themselves. Exp) Option d is the correct answer.
According to the Panchayats (Extension to Scheduled
35. Solution: (a)
Areas) Act, 1996 or PESA, which is a law enacted by the
Exp) Option a is the correct answer. Government of India for ensuring self-governance through
The alteration in the limits of Block and Zila Panchayats traditional Gram Sabha's for people living in the Scheduled
is notified by the Governor of the respective state. When Areas of India, the meeting of Gram Sabha in Scheduled
there is a need to alter the boundaries or geographical Areas is presided over by a Schedule Tribe member present
limits of Block and Zila Panchayats, it is usually done to who is elected by Gram Sabha.

PYQ W o r k b o o k - - - - - - - - - - - - - - - - - - - - - - - - - - - - - - - - - - - - - 1 1 1

KING R QUEEN P [ऋषभ राजपूत ]


II II
39. Solution: (b) 42. Solution: (b)
Exp) Option b is the correct answer Exp) Option bis the correct answer.
Statement l is correct: In the Panchayati Raj System, all As per the 73rd Amendment to the Constitution of India,
seats in a Panchayat are filled through direct elections. The individuals must be at least 21 years of age to be eligible for
members of the Panchayat, including the Sarpanch (village Panchayat membership, contrary to the mentioned 25 years.
head) or Chairperson, are elected by the people through the This amendment lowered the age requirement to encourage
process of direct voting. youth participation in local governance.

Statement 2 is correct: The 'Gram Sabha' is a village-level Important Tips


assembly that consists of all the adults who are registered in Panchayati Raj was given constitutional status through
the electoral rolls of the village. The members of the Gram the 73rd Constitutional Amendment Act, 1992 (Part
Sabha have the right to participate in the decision-making IX, Schedule 11, Article 243-243 0), and it was first
process of the Panchayat and are involved in various local established in Rajasthan on October 2, 1959, followed
governance activities. by Andhra Pradesh, as recommended by the Balwant Rai
Mehta Committee.
Statement 3 is correct: The election of the Chairperson of
The 73rd Constitutional Amendment Act aims to provide
a Panchayat is governed by the respective state's legislation.
a three-tier system of Panchayati Raj, which consists of
Each state in India has its own law that defines the process
Village-level Panchayats, Block-level Panchayats, and
and criteria for electing the Chairperson of a Panchayat. District-level Panchayats . Panchayati Raj System (PRS)
Statement 4 is incorrect: While most states in India have a was initially implemented based on the recommendations
three-tier system of Panchayats, there are a few exceptions. of the Balwant Rai Mehta Committee. Later, it was granted
Some states have a two-tier system, consisting of the Gram constitutional status through on recommendation of the
Panchayat at the village level and the Zilla Parishad at the LM Singhvi committee.
district level. States like Kerala and Goa follow a two-tier Different committees associated with Panchayati Raj (in
system, whereas states like Rajasthan, Maharashtra, and Chronological Order):
Andhra Pradesh have a three-tier system. Balwant Rai Mehta Committee (January 1957)
Ashok Mehta Committee (December, 1957)
40. Solution: (d)
G.V.K. Rao Committee (1985)
Exp) Option d is the correct answer
L.M. Singhvi Committee (1986)
Statement l is correct: The elections to the Panchayats
Thungon Committee (1988)
in India are conducted by the respective State Election
Commissions (SECs). The State Election Commission is Gadgil Committee ( 1988)
responsible for overseeing and conducting the electoral The Panchayati Raj was implemented under the Prime
process for Panchayat elections within the state under Ministership of P. V. Narasimha Rao.
Article 243K of the constitution. 43. Solution: (b)
Statement 2 is incorrect: The State Election Commissioner Exp) Option b is the correct answer.
is appointed by the Governor of the state.
Each Panchayat operates under its own set of regulations,
Statement 3 is correct: As per the provisions of the with the state legislature conferring powers to facilitate
Constitution of India, the State Legislatures have the power their functioning effectively. These powers encompass
to make laws on all matters relating to Panchayat elections. formulating economic development and social justice plans,
This includes the conduct of elections, the qualifications plan implementation, and handling matters specified in the
and disqu~ lifications of candidates, the delimitation of eleventh schedule.
constituencies, and other related aspects.
44. Solution: (b)
41. Solution: (a) Exp) Option b is the correct answer.
Exp) Option a is the correct answer. The 11th Schedule of the Constitution of India distributes
While MGNREGA guarantees 100 days of wage employment, powers between the State Legislatures and the Panchayats.
These powers include areas such as rural development,
it primarily focuses on providing unskilled manual work
planning, agriculture, education, health, water supply,
rather than specifically emphasizing skilled labor.
sanitation, poverty alleviation, and social justice. The
The other objectives of MGNREGA include the creation State Legislatures are responsible for enacting laws and
of productive assets, enhancing livelihood security, and providing the necessary framework for the functioning of
ensuring empowerment to women. These objectives aim to the Panchayats, while the Panchayats have the authority to
enha11ce livelihood security of households in rural areas of implement and execute various developmental programs and
the country. initiatives at the grassroots level.

fDr--------------------------------------PYQWorkbook
laforumlm

KING R QUEEN P [ऋषभ राजपूत ]


II INDIAN POLITY A:l'iD QClVERNANCE II
45. Solution: (b) 47. Solution: (c)
Exp) Option b is the correct answer. Exp) Option c is the correct answer.
The elected members of the District Council shall hold Article 243 G mentions "subjected to the provision of
office for a term of five years from the date appointed this constitution the legislature of a state which says that
for the first meeting of the Council after the general Panchayats are under the umbrella of the Constitution.
elections to the Council. (option bis correct)
Within the constitutional framework, 'Panchayats' is a
Sixth schedule to the constitution of India provides for State subject. Hence, the Panchayat does not work on
setting up of autonomous district council the principle of constitutional autonomy. (Statement 3
The Constitution of India makes special provisions for is not correct)
the administration of the tribal dominated areas in four
48. Solution: (d)
states viz Assam, Meghalaya Tripura and Mizoram.
Exp) Option d is the correct answer.
As per Article 244 and sixth Schedule these areas are
called "Tribal Areas The Constitution of India does not contain any provision
The Sixth Schedule envisages establishment of for the constitution of urban panchayats in every state. The
Autonomous District Councils (ADC) Constitution only makes provisions for the constitution
of municipalities, which are divided into three categories-
Important Tips Nagar Panchayats, Municipal Council, and Municipal
District council for each autonomous district: Corporations. The Constitution leaves it up to the states to
It has 30 members. decide whether or not to constitute urban panchayats.
4 are nominated by the governor - They perform their 49. Solution: (c)
duties during the pleasure of the governor
Exp) Option c is the correct answer.
26 are elected using the adult franchise - Their term
Statement 1 is incorrect- According to the Article 243G
of office is five years.
of the Indian Constitution, the state legislature has the
There is a separate regional council for each
authority to endow Panchayats with powers and authority
autonomous district
necessary for self-government. The state legislature may
• The laws related to the following can be made by the authorize a Panchayat to levy, collect, and appropriate taxes,
regional and autonomous councils with the assent duties, tolls, and fees.
of the governor: Land, Forest, Canal Water, Shifting
Cultivation, Village Administration, Inheritance of Statement 2 is correct- The Constitution empowers the
Property, Marriage & Divorce, Social Customs. State Legislature to devolve powers to Panchayats for the
preparation of plans related to economic development and
• The central and state acts do not apply on these
social justice. This enables Panchayats to play a role in local-
autonomous and regional councils (unless modified
level planning and implementation of development schemes.
and accepted).
Statement 3 is correct- The State Legislature can also
46. Solution: (b) devolve powers to Panchayats for the implementation of
Exp: Option b is the correct answer. schemes related to economic development and social
Part IX of the 73rd Constitutional amendment act 1992 justice. This enables Panchayats to actively participate in the
constitutionalised the Panchayati Raj Institutions (PRis) execution of development programs at the local level.
Article 243, 243A to 243 0 under Part IX has been 50. Solution: (b)
included in the Indian constitution.
Exp) Option b is the correct answer
The PRis are the local self-governing bodies that
Articles 371 to 371- J in Part XXI of the Constitution ofindia
ensure the opportunity for people's participation and
contain special provisions for twelve states: Maharashtra,
involvement in the formulation and implementation of
Andhra Pradesh, Telangana, Sikkim, Mizoram, Arunachal
rural development programmes.
Pradesh, Gujarat, Nagaland, Assam, Manipur, Goa and
The main objective of Panchayat System in India is to Karnataka. Madhya Pradesh is not mentioned in Article
strengthen the base of democracy at the grass root level. 371 of the Constitution of India.
Part IX A of the 74th Constitutional amendment act 1992
constitutionalised the The Municipality in the Indian 51. Solution: (c)
constitution. Exp) Option c is the correct answer
• Few states like Nagaland, Meghalaya, Mizoram and some In India, the power to levy taxes by Panchayats is primarily
other tribes don't have a three tier system of government. within the domain of the State Governments. The State
The act doesn't apply to these areas. (statement 1 is not Governments have the authority to authorize Panchayats to
correct) levy certain taxes, fees, or other local charges.

PYQ W o r k b o o k - - - - - - - - - - - - - - - - - - - - - - - - - - - - - - - - - - - - - - 1 & 1
mForum~'8

KING R QUEEN P [ऋषभ राजपूत ]


II .INDIAN P()LITYANDGOYERN.ANCE II
This means that he/ she has to examine whether the 56. Solution: (d)
expenditure is in accordance with the rules and regulations, Exp) Option d is the correct answer
as well as whether it is prudent, economical, and efficient.
The SHRC does not have the power to punish for the
Reason (R) is correct: He/she has to uphold the Constitution violation of human rights. This power is vested with the
and the laws of Parliament in the field of financial courts. The SHRC can only recommend measures to the
administration. This means that he/she has to ensure that state government to improve the protection of human rights
the executive (i.e., the council of ministers) is accountable to in the state.
Parliament in the area of financial administration. The CAG's
57. Solution: (d)
role is not limited to verifying the legality of expenditures but
also extends to upholding the constitutional and legislative Exp) Option d is the correct answer
provisions related to financial administration. The State Public Service Commission is not consulted on
making transfers of Civil Servants because transfers are
53. Solution: (c) considered to be administrative decisions falling under
Exp) Option c is the correct answer the purview of the executive branch of the government.
The Central Vigilance Commission (CVC) was established The State Public Service Commission's role is primarily
limited to matters related to recruitment, appointments,
in 1964 on the recommendations of the Santhanam
promotions, and disciplinary matters.
Committee, which was set up in 1962 to review the working
of the central government's anti-corruption machinery. It 58. Solution: (c)
had suggested a high-powered central agency to investigate Exp) Option c is the correct answer.
corruption cases involving government employees and
The CAG can be removed from his/her office in like manner
recommend disciplinary action against corrupt officials.
as a judge of the Supreme Court, who can also be removed
Important Tips by the President only on an address from both Houses of
Parliament on the grounds of proved misbehaviour or
The Gorwala Committee (1951) recommendations to
incapacity under Article 124 (4) of the Constitution.
enhance India's public administration:
The committee proposed changes in the recruitment 59. Solution: (d)
process for administrators to ensure efficiency and Exp) Option dis the correct answer
competence in the civil services. The Finance Commission consists of a chairman and four
Emphasis was placed on improving training to equip other members to be appointed by the president. Hence, there
administrators with necessary skills for effective are total 5 members in commission. According to Article
governance. 280 (2) of the constitution, Parliament may by law determine
the qualifications which shall be requisite for appointment as
Mechanisms were suggested to strengthen the
members of the Commission and the manner in which they
connection between administrators and legislative
shall be selected. Hence, it is not necessary that at least one
bodies for improved coordination and governance. member of the Commission must be a Judge of the High
Recommendations aimed at establishing a framework Court or Supreme Court. As per Section 6 in The Finance
for administrators to work closely with the Planning Commission (Miscellaneous Provisions) Act, 1951, every
Commission for planned economic development. member of the Commission shall hold office for such period
Setting up of Organisation & Method (0 & M) as may be specified in the order of the President appointing
him, but shall be eligible for reappointment: Provided that
divisions at various government levels was suggested
he may, by letter addressed to the President, resign his office.
to enhance organizational efficiency.
The First Chairman of the Commission was K. C. Neogy
54. Solution: (a) and Fifteenth Chairman is N. K. Singh.
Exp) Option a is the correct answer 60. Solution: (c)
Administrative Reforms Commission of India (1966-70) Exp) Option c is the correct answer.
was headed by Morarji Desai, and it submitted a report that Statement I is incorrect- Out of the 543 elected seats in the
recommended the establishment of Lokpal and Lokayukta Lok Sabha, 84 are reserved for Scheduled Castes and 47
at the central and state levels. The commission argued that are reserved for Scheduled Tribes. The allocation of seats
these institutions would help to improve the standards of to them is determined based on their percentage of the total
public administration and to prevent corruption. population in the country. This ensures that the reservation
system does not exclude representation for other social
55. Solution: (a)
groups in the country.
Exp) Option a is the correct answer
Statement II is correct- The Delimitation Commission
According to Article 316( 1) of the Indian Constitution, is responsible for deciding which constituencies are to be
the Chairman and members of the Union or Joint Public reserved for Scheduled Castes (SCs) and Scheduled Tribes
Service Commission are appointed by the President. (STs).

PYQ W o r k b o o k - - - - - - - - - - - - - - - - - - - - - - - - - - - - - - - - - - - - - - -

KING R QUEEN P [ऋषभ राजपूत ]


II II
Statement III is incorrect- The Delimitation Commission Prosecution of Offenders: Ensure swift prosecution of
is appointed by the President of India and operates in individuals involved in criminal activities, especially
coordination with the Election Commission of India.
those connected to politicians and government
Important Tips officials.
In India, Delimitation Commissions have been constituted Strengthen Criminal Justice System: Enhance legal
on four occasions: in 1952 under the Delimitation provisions to effectively combat economic offenses and
Commission Act, 1952; in 1963 under the Delimitation organized crime.
Commission Act, 1962; in 1973 under the Delimitation Act,
1972; and in 2002 under the Delimitation Act, 2002. The Focus on Criminal Gangs: Direct law enforcement
Delimitation Commission consists of a retired Supreme to target not only individual crimes but also criminal
Court judge, the Chief Election Commissioner, and the gangs, armed groups, drug mafias, and smuggling
respective State Election Commissioners. The primary
functions of the Delimitation Commission are as follows:
networks.
End Political Patronage: Sever ties between
j
Determining the boundaries of territorial politicians and criminal elements, breaking the cycle
constituencies in the country to account for changes
of criminalization of politics,
in population. ~------------·--- ----·-·-~-··----···~---~

Identifying seats reserved for Scheduled Castes and 64. Solution: (d)
Scheduled Tribes in areas where their population is
Exp) Option d is the correct answer.
relatively high.
The CAG submits his/her reports relating to the accounts
61. Solution: (c) of the Union to the President, who shall cause them to be
Exp) Option c is the correct answer. laid before each House of Parliament under Article 151 ( 1) of
Article 326 of the Constitution of India recognizes adult the Constitution. Similarly, the CAG submits his/her reports
suffrage. It states that every citizen of India who is 18 years relating to the accounts of a State to the Governor, who shall
of age or above on the qualifying date is entitled to be cause them to be laid before the Legislature of the State
registered as a voter at any election to the Lok Sabha and under Article 151 (2) of the Constitution.
the State Legislative Assemblies.
Important Tips
Important Tips
The Comptroller and Auditor General of India (CAG)
Article 324-- Superintendence, direction and control of
is the supreme audit authority of the country and
elections to be vested in an Election Commission.
he/she audits the accounts of the Union and State
Article 325- No person to be ineligible for inclusion in, governments.
or to claim to be included in a special, electoral roll on
He/she is appointed by the President of India under
grounds of religion, race, caste or sex.
Article 148 of the Constitution and holds office for a
Article 327- Power of Parliament to make provision with term of six years or until he/ she attains the age of 65
respect to elections to legislatures. years, whichever is earlier.
62. Solution: (c) He/she can be removed by the President only on an
Exp) Option c is the correct answer address from both Houses of Parliament on the grounds
of proved misbehaviour or incapacity
The doctrine of pleasure in Article 310 of the Indian
Constitution allows the government to terminate the 65. Solution: (a)
employment of public servants at will. It grants the
Exp) Option a is the correct answer
government the right to appoint and dismiss public servants.
According to Article 316(1) of the Indian Constitution, the
63. Solution: (c) Chairman and other members of a State Public Service
Exp) Option c is the conect answer Commission are appointed by the Governor of the
The Vohra (Committee) Report was submitted by the former respective state.
Indian Home Secretary, N. N. Vohra, in October 1993. It
studied the problem of the criminalization of politics and 66. Solution: (b)
of the nexus among criminals, politicians and bureaucrats Exp) Option bis the correct answer.
in India.
The power to grant or refuse leave to the CAG and to
Important Tip revoke or curtail leave granted to him/her, shall vest
Key Recommendations of Vohra Committee: in the President, as per Section 5 (1) of the Comptroller
Establishment of a Nodal Point: Create a central hub and Auditor General's (Duties, Powers and Conditions of
for intelligence and enforcement agencies to promptly Service) Act, 1971.
share crime syndicate-related information, improving CAG is appointed by the President oflndia under Article 148
coordination. of the Constitution and holds office for a term of six years or

1 1 1 1 - - - - - - - - - - - - - - - - - - - - - - - - - - - - - - - - - - - - - - P Y Q Workbook
II Foru n,1118

KING R QUEEN P [ऋषभ राजपूत ]


II INDIAN l'OLITYAND. GQV:EllNANCE II
until he/she attains the age of 65 years, whichever is earlier. 70. Solution: (b)
He/she can be removed by the President only on an address Exp) Option b is the correct answer
from both Houses of Parliament on the grounds of proved The Chairman of the Council of States is not a member
misbehavior or incapacity. of the committee. Section 2, 3 and 4 of The Protection of
67. Solution: (b) Human Rights Act, 1993 lay down the rules for appointment
to the NHRC. The Chairperson and members of the
Exp) Option b is the correct answer.
NHRC are appointed by the President of India, on the
Statement 1 is correct: It is the duty of the Comptroller and recommendation of a committee consisting of:
Auditor-General (CAG) to audit all receipts that are payable
The Prime Minister (Chairperson)
into the Consolidated Fund of India. The CAG ensures that
the revenue received by the government is properly audited The Home Minister
and accounted for. The Leader of the Opposition in the Lok Sabha (Lower
Statement 2 is correct: The CAG has the authority to audit House)
and report on the accounts of stores or stock kept in any • The Leader of the Opposition in the Rajya Sabha (Upper
office or department of the Union or a State. This authority House)
allows the CAG to examine the management and utilization The Speaker of the Lok Sabha (Lower House)
of stores or stock maintained by government offices or
The Deputy Chairman of the Rajya Sabha (Upper House)
departments.
Important Tips
68. Solution: (d)
The Protection of Human Rights (Amendment) Act,
Exp) Option d is the correct answer 2019
Governor is not member of the committee to recommend NHRC Composition: The Act allowed a former Chief
name for the appointment of Chairperson and members of Justice or Judge of the Supreme Court to become
State Human Rights Commission. Instead, the Chairperson NHRC's chairperson. The number of members
and members of the SHRC are appointed by the Governor, increased from two to three, with at least one member
on the recommendation of a committee consisting of: being a woman. Chairpersons of commissions for
The Chief Minister (chairperson) Backward Classes, Child Rights, and Disabilities would
also join NHRC as members.
The Home Minister
The Leader of the Opposition in the legislative council SHRC Chairperson: The Bill allowed a former Chief
Justice or Judge of a High Court to become the
The Leader of the Opposition in the legislative assembly chairperson of SHRC.
The Speaker of the legislative assembly Term of Office: The term of office for NHRC and
The Chairman of the legislative council SHRC chairpersons and members was reduced to
three years or until seventy years of age. The limit on
69. Solution: (d)
reappointment for members was removed.
Exp) Option d is the correct answer. Powers of Secretary-General: The Secretary-General
Article 149 of the Indian Constitution prescribes the duties and Secretary of NHRC and SHRCs received broader
and powers of the Comptroller and Auditor General administrative and financial powers, excluding judicial
( CAG) ofindia. This article outlines the role, functions, and functions, under the respective chairperson's control.
responsibilities of the CAG in auditing and reporting on the Union Territories: The central government could
accounts of the Union and the States. It provides the CAG assign human rights functions of Union Territories to
with the authority to examine and audit the accounts related SHRCs, while Delhi's human rights functions would
to the receipt and expenditure of public funds. remain under NHRC's jurisdiction.
Important Tips 71. Solution: (a)
Article 146 deals with the officers and servants and the Exp) Option a is the correct answer
expenses of the Supreme Court. It states that the Chief
Governor of the state appoints the chairperson and other
Justice of India can appoint officers and servants of the
members of the State Human Rights Commission.
Supreme Court in consultation with the President.
Article 147 covers the interpretation of the constitution Important Tips
along with that of the government of India act, 1935. State Human Rights Commission:
Article 148 states that there shall be a CAG who shall be Composition: The Commission is made up of three
appointed by the President by warrant under his hand and members: a chairperson and two members.
seal and shall only be removed from office in like manner Chairperson: The chairperson shall be a retired Chief
and on like grounds as a Judge of the Supreme Court. Justice of a High Court or a High Court Judge

PYQ W o r k b o o k - - - - - - - - - - - - - - - - - - - - - - - - - - - - - - - - - - - - - 1 1 1

KING R QUEEN P [ऋषभ राजपूत ]


II fNDI:ANPOLITY :ANOGOVERl'.\JANCE II
Members: Members should be a serving or retired High Section 175, Section 178, Section 179, Section 180 or Section
Court judge or a District Judge. With a minimum of 228 of the Indian Penal Code, the Human Rights Commission
seven year experience as a District Judge, as well as is to be deemed a civil court. This means that the Human
someone with knowledge or practical experience in Rights Commission has the same powers as a civil court in
human rights. respect of these offences. For example, the Human Rights
Tenure: The chairperson and members are elected for Commission can issue summons, compel the attendance of
a three-year term or until they reach the age of 70, witnesses, and take evidence on oath.
whichever comes first. Important Tips
Removal: The governor appoints the chairperson Sections of IPC mentioned in the question:
and members of a State Human Rights Commission,
Section 175:- Omission to produce document to public
however, they can only be removed by the President
servant by person legally bound to produce it.
72. Solution: (b) Section 178:- Refusing oath or affirmation when duly
Exp) Option b is the correct answer required by public servant to make it.
The Chairperson of the Law Commission of India is not Section 179:- Refusing to answer public servant
the ex-officio member of the National Human Rights authorized to question.
Commission. Section 180:- Refusing to sign statement.
Ex-officio members of NHRC: Section 228:- Intentional insult or interruption to
Chairperson, National Commission for Scheduled Castes public servant sitting in judicial proceeding.
Chairperson, National Commission for Scheduled Tribes 77. Solution: (c)
Chairperson, National Commission for Minorities Exp) Option c is the correct answer
Chairperson, National Commission for Women
The Protection of Human Rights Act, 1993 (PHRA) provides
Chairperson, National Commission for Backward Classes that while dealing with complaints of violation of Human
Chairperson National Commission for Protection of Rights by the members of the armed forces, the Commission
Child Rights either on its own motion or on receipt of a petition, shall
Chief Commissioner for Persons with Disabilities seek a report from the Central Government. The Central
Government shall, within a period of three months, or
73. Solution: (b)
such longer period as may be specified by the Commission,
Exp) Option b is the correct answer submit a report to the Commission. The Commission shall,
A Sitting Judge of the High Court or a Sitting District Judge after considering the report of the Central Government,
can be appointed as a member of the State Human Rights take such action as it deems fit.
Commission after consultation with the Chief Justice of
the High Court of the concerned State. 78. Solution: (d)
Exp) Option d is the correct answer
74. Solution: (b)
The objective of the Protection of Human Rights Act, 1993,
Exp) Option b is the correct answer
was to achieve all of the following:
The State Human Rights Commission (SHRC) submits its
annual or special reports to the state government. These The primary aim of the Act was to ensure improved
reports are laid before the state legislature, along with a protection and promotion of human rights in India.
memorandum of action taken on the recommendations of The Act established the National Human Rights
the Commission and the reasons for nonacceptance of any of Commission (NHRC) at the national level to inquire into
such recommendations. human rights violations and recommend measures for
their prevention and redressal.
75. Solution: (b)
Exp) Option b is the correct answer Apart from the NHRC, the Act also allowed for the
creation of State Human Rights Commissions (SHRCs)
Section 39 of the Protection of Human Rights Act, 1993 says
in each state to address human rights issues at the state
that Every Member of the Commission, State Commission
level.
and every officer appointed or authorized by the Commission
or the State Commission to exercise functions under this Act Important Tips
shall be deemed to be a public servant.
key provisions of the Protection of Human Rights Act,
76. Solution: (b) 1993:
Exp) Option b is the correct answer The establishment of the NHRC and SHRCs
As per the section 13 ( 4) of the national human rights The investigation of allegations of human rights
protection act, 1993, In relation to offences described under violations

1 1 1 , _ _ _ - - - - - - - - - - - - - - - - - - - - - - - - - - - - - - - - - - - - P Y Q Workbook
GIFonmi~

KING R QUEEN P [ऋषभ राजपूत ] •.


II I:NDIAN POLITYAND GOVERNANCE II
The taking of appropriate action against those It consists of only two articles: article 323 A dealing
responsible for human rights violations with administrative tribunals and article 323 B
The establishment of Human Rights Courts dealing with tribunals for other matters.

The training of law enforcement officials on human Article 323 A empowers the parliament to establish
the administrative tribunals in 1985.
rights
This article empowers the Central government to
The promotion of human rights education
establish one central administrative tribunal and the
79. Solution: (c) state administrative tribunals.
Exp) Option c is the correct answer. Article 323 B empowers the parliament and state
legislature to establish tribunal for different disputes
National Voters Day is celebrated every year on 25 January
in India to mark the foundation of the Election Commission 83. Solution: (a)
of India. The Election Commission of India was established Exp) Option c is the correct answer.
on 25 January 1950. Established in: 1948
Important Tip Used for: immediate relief to families of those killed in
The main objective of celebrating National Voters Day is to natural calamities like floods, cyclones and earthquakes,
encourage people to participate in the electoral process and etc. and to the victims of the major accidents and riots
to make them aware of their rights and responsibilities Headed by: Prime Minister
as voters. On this day, various awareness programmes Administration: Administered on an Honorary basis by
are organized by the Election Commission of India and Joint Secretary to the Prime Minister as Secretary of the
other organizations to promote voter education and fund.
participation. PMNRF operates from the Prime Minister's Office
(PMO) (option a is correct)
80. Solution: (b)
Features:
Exp) Option b is the correct answer
(a) Consists entirely of public contributions and does not get
The Telecom Regulatory Authority of India (TRAI)
any budgetary support.
is responsible for the implementation of the
(b) Recognized as a Trust under the Income Tax Act
Government of India's policy in all matters concerning
telecommunications in India. TRAI is an independent (c) Exempt under the Income Tax Act, 1961 under Section
regulatory body established under the Telecom Regulatory 10 and 139 for return purposes.
Authority oflndia Act, 1997. It functions as the regulator for 84. Solution: (d)
the telecommunications sector in India and has the authority
Exp) Option d is the correct answer.
to make regulations, issue guidelines, and ensure compliance
with the government's policies in the telecom sector. The Commission shall, perform all or any of the following
functions, namely:-
81. Solution: (b) Inquire at its own initiative on the violation of human
Exp) Option bis the correct answer rights
Article 338A of the Indian Constitution deals specifically Inquiry on a petition presented to it by a victim
with the establishment of the National Commission for Visit to jails to study the condition of the inmates
Scheduled Tribes (NCST). This article was inserted into Undertaking and promoting research in the field of
the Constitution through the 89th Amendment Act of 2003. human rights
The NCST is a constitutional body tasked with safeguarding encourage the efforts of NGOs and institutions working
the rights and interests of the Scheduled Tribes in India. in the field of human rights
82. Solution: (c) review the factors, including acts of terrorism that inhibit
the enjoyment of human rights
Exp) Option c is the correct answer.
such other functions as it may consider necessary for the
Tribunals established by a law of the Parliament can exclude promotion of human rights
the jurisdiction of all courts (except the jurisdiction of
Supreme Court under Article 136) to allow to special leave Important Tips
to appeal. It is formed as per the statutory prov1s10ns of
Protection of Human Rights Act, 1993.
Important Tips
It is a multi-member body consist of Chairman and
The dictionary meaning of the word 'Tribunal' is seat four members. President appoints the Chairman
of the Judge. and members of the NHRC on the recommendation
The 42nd amendment act of 1976 added a new part of the high-powered committee headed by the Prime
XIV-A entitled as Tribunals. Minister.

PYQ W o r k b o o k - - - - - - - - - - - - - - - - - - - - - - - - - - - - - - - - - - - - - - - 1 & 1

KING R QUEEN P [ऋषभ राजपूत ]

You might also like